澳洲AMC等级D 9-10年级全英文完整版

Download as pdf or txt
Download as pdf or txt
You are on page 1of 126

a selection of Australian Mathematics Trust publications

Indicate Quantity Required in Box


AUSTRALIAN MATHEMATICS COMPETITION BOOKS
2007 AMC Solutions and Statistics Primary Version – $A35.00 each A u s t r a l i a n M a t h e ma t i c s C o m p e t i t i o n
2007 AMC Solutions and Statistics Secondary Version – $A35.00 each f o r t h e W e s t p a c A wa r d s
2007 AMC Solutions and Statistics primary and Secondary Versions – $A57.00 for both
Two books are published each year for the Australian Mathematics Competition for the Westpac Awards, a Primary version for the
Middle and Upper Primary divisions and a Secondary version for the Junior, Intermediate and Senior divisions. The books include an activity of the australian mathematics trust
the questions, full solutions, prize winners, statistics, information on Australian achievement rates, analyses of the statistics as well
as discrimination and difficulty factors for each question. The 2007 books will be available early 2008.
Australian Mathematics Competition Book 1 (1978-1984) – $A40.00 each
W e d n e s d ay 2 5 J u l y 2 0 07
Australian Mathematics Competition Book 2 (1985-1991) – $A40.00 each
Australian Mathematics Competition Book 3 (1992-1998) – $A40.00 each
Australian Mathematics Competition Book 3-CD (1992-1998) – $A40.00 each
intermediate Division Competition Paper
Australian Mathematics Competition Book 4 (1999-2005) – $A40.00 each
These four books contain the questions and solutions from the Australian Mathematics Competition for the Westpac australian School Years 9 and 10
Awards for the years indicated. They are an excellent training and learning resource with questions grouped into topics
and ranked in order of difficulty.
time allowed: 75 minutes
BOOKS FOR FURTHER DEVELOPMENT OF MATHEMATICAL SKILLS
Problems to solve in middle school mathematics – $A50.oo each
This collection of problems is designed for use with students in Years 5 to 8. Each of the 65 problems is presented ready to
be photocopied for classroom use. With each problem there are teacher’s notes and fully worked solutions. Some problems I n s t r uc t i o n s a n d I n f o r m at i on
have extension problems presented with the teacher’s notes. The problems are arranged in topics (Number, Counting, Space
and Number, Space, Measurement, Time, Logic) and are roughly in order of difficulty within each topic. GENERAL
Problem Solving via the AMC – $A40.00 each 1. Do not open the booklet until told to do so by your teacher.
This book uses nearly 150 problems from past AMC papers to demonstrate strategies and techniques for problem solving.
The topics selected include Geometry, Motion and Counting Techniques. 2. NO calculators, slide rules, log tables, maths stencils, mobile phones or other calculating aids are
Challenge! (1991–1995) – $A40.00 each permitted. Scribbling paper, graph paper, ruler and compasses are permitted, but are not essential.
This book reproduces problems, solutions and extension questions from both Junior (Years 7 and 8) and Intermediate 3. Diagrams are NOT drawn to scale. They are intended only as aids.
(Years 9 and 10) versions of the Mathematics Challenge for Young Australians, Challenge Stage. It is a valuable resource
book for the classroom and the talented student. 4. There are 25 multiple-choice questions, each with 5 possible answers given and 5 questions that
The above prices are current to 31 December 2007. Details of other AMT publications are available on the Australian
require a whole number between 0 and 999. The questions generally get harder as you work
Mathematics Trust’s web site www.amt.canberra.edu.au/amtpub.html. through the paper. There is no penalty for an incorrect response.
ALL BOOKS CAN BE ORDERED ONLINE @ www.amt.edu.au/amtpub.htnl
5. This is a competition not a test; do not expect to answer all questions. You are only competing against
payment details Payment must accompany orders. Please allow up to 14 days for delivery. your own year in your own State or Region so different years doing the same paper are not compared.
Please forward publications to: (print clearly) 6. Read the instructions on the Answer Sheet carefully. Ensure your name, school name and school year
name: are filled in. It is your responsibility that the Answer Sheet is correctly coded.
address: 7. When your teacher gives the signal, begin working on the problems.

THE ANSWER SHEET


COUNTRY: postcode:
Postage and Handling - within Australia, add $A3.00 for the first book and $A1.00 for each additional book
1. Use only lead pencil.
- outside Australia, add $A13.00 for the first book and $A5.00 for each additional book 2. Record your answers on the reverse of the Answer Sheet (not on the question paper) by FULLY
total: Cheque/Bankdraft enclosed for the amount of $A colouring the circle matching your answer.
Please charge my Credit Card (Visa, Mastercard) Amount authorised:$A 3. Your Answer Sheet will be read by a machine. The machine will see all markings even if they are in the
Cardholder’s Name (as shown on card): wrong places, so please be careful not to doodle or write anything extra on the Answer Sheet. If you
Cardholder’s Signature: want to change an answer or remove any marks, use a plastic eraser and be sure to remove all marks and
Expiry Date: Date: Tel (bh): smudges.

Card Number: INTEGRITY OF THE COMPETITION


All payments (cheques/bankdrafts, etc) should be in Australian currency, made payable to The AMC reserves the right to re-examine students before deciding whether to grant official status to their
Australian Mathematics Trust and sent to: score status to their score.
Australian Mathematics Trust, University of Canberra ACT 2601, Australia.
Tel: 02 6201 5137 Fax: 02 6201 5052

©AMT Publishing 2007 amtt limited acn 083 950 341


Intermediate Division

Questions 1 to 10, 3 marks each

8×9
1. equals
3
(A) 27 (B) 11 (C) 24 (D) 20 (E) 17

..
...
...
..
.
...
.... ...
....
.... ...
...
2. In the diagram, where P QR is a straight
.... .
....
.... ...
.... ...
..... ..
..
line, x equals
.... .
.
....
...
x◦
....
....
.... ...
.... ..
.... ...

75◦
.... .
25 .... ...
(A) 60 (B) 70 (C) 80 (D) 90 (E) 100 ....
.....
.... ....
..
..
......
.

P Q R

3. If 110 + x = 97 + y, then

(A) x + 13 = y (B) x = y + 13 (C) x + y = 13


(D) x + y = 207 (E) x − y = 207

4. Andy bought 2 chocolates at $1.35 each. How much change should he get from $5?

(A) $2.70 (B) $2.60 (C) $3.30 (D) $2 (E) $2.30

5. Of the following, which is the largest fraction?


7 3 6 4 1
(A) (B) (C) (D) (E)
15 7 11 9 2

6. The average weight of a group of 4 boys and 6 girls is 64 kg. The average weight
of the boys is 70 kg. What is the average weight of the girls?

(A) 58 kg (B) 59 kg (C) 60 kg (D) 61 kg (E) 62 kg


I2
7. Nicky started a mobile phone call at 10:57 am. The charge for the call was 89 cents
per minute and the total cost for the call was $6.23. Nicky’s call ended at

(A) 11:27 am (B) 11:14 am (C) 11:04 am (D) 11:46 am (E) 11:05 am

.... .......
....
... .......
8. A fish tank in the shape of a rectangular prism is ....
....
....
.... ...
...
....
....
... ... ...
1 m long and 25 cm wide. If it holds 55 L when full, ...
...
its height, in centimetres, is ...... .... .... .... .... .... .... .... .... .... .... .... .... .... .... .... ........
....
..
...
....
.... ....
... ...
....
(A) 11 (B) 22 (C) 44 (D) 110 (E) 220 ....
.
...

9. The larger of two numbers is 3 more than twice the smaller number. If their sum
is 18, what is the smaller number?

(A) 3 (B) 4 (C) 5 (D) 7 (E) 9

...
........
...........
..............
.
...........................
.
.......................
10. A square with side length 2 units is placed ..........................
.............................
...... . . . . . . . . . . . . .
...
...................................................................
next to a square with side length 5 units as ...... . . . . . . . . . . . . . . . .
..........................................
..............................................
..................................................
...............................................................................
shown. The shaded area, in square units, is ...
. .........................
....... . . . . . . . . . . . . . . . . . . . . . . . . . .
............................................................
....... ......................................................... 5
....................................................................
..........................................................................................................................................
..
.........................................................................
(A) 13.5 (B) 14.5 (C) 18.5 ...........................................................................
............................................................................
............................................................................................................................................................
..

(D) 19.5 (E) 26 2 .


... . . . . . . . . . . . . . . . . . . . . . . . . . . . . . . . . . . . . . . .
...................................................................................
......................................................................................
.......................................................................................
.
........................................................................................................................................
... .........................................................................................
... . . . . . . . . . . . . . . . . . . . . . . . . . . . . . . . . . . . . . . . . . . . . .

Questions 11 to 20, 4 marks each

11. Successive discounts of 10%, 20% and 50% are equivalent to a single discount of

(A) 64% (B) 75% (C) 26 23 % (D) 36% (E) 70%

12. The game of Four Tofu is played on a 4 × 4 grid. 2


When completed, each of the numbers 1, 2, 3 and
4 occurs in each row and column of the 4 × 4 grid 1
and also in each 2 × 2 corner of the grid. 1 3
When the grid shown is completed, the sum of the 4
four numbers in the corners of the 4 × 4 grid is
(A) 13 (B) 11 (C) 15 (D) 12 (E) 10
I3

13. In the diagram,  P OX =  QOX and  QOY =  Y OZ =  ZOR. If  P OY = 33◦


and  XOZ = 45◦ , the size of  P OR is

Y... Z....... R .
..
..
Q...... ...
...
...
...
...
...
...
..
..
..
..... . .. .
.
X ..........
...........
..... ... ... ..
.... .... .. .
.
.......... ............. .... ...
.
........... ....... .. ..
....... .........
P ........................................................................

(A) 60◦ (B) 65◦ (C) 69◦ (D) 71◦ (E) 78◦

14. P QRS is a parallelogram and T lies on P .. ......


T Q
. ..... ... ...
... ..... .. ...
.....
P Q such that P T : T Q = 3 : 2. The .
..
...
....
.....
..
.....
..
..
... .
...
...
..
.. .... .. ..
....
ratio of the area of P T RS to the area
..
... .... .. ...
.... .....
. .. .
...
... .. .
.. ..... .. ..
... .. ...
of P QRS is ...
..
. ..
......
....
..... ..
..
..
..
..
.
.
... .... ... ...
.. .... ..
.. .... .. ...
.. .... .. ..
..... ..
(A) 1 : 2 (B) 2 : 3 (C) 3 : 4 ....
.. .....
... ..
. ..
..
.. ....
.
.
... .... .. ..
.. .... .. ..
.. .... .. ..
......
(D) 4 : 5 (E) 5 : 6 S R
...

15. When 50 is divided by a whole number, the remainder is 5. How many different
values are possible for this whole number?

(A) 1 (B) 2 (C) 3 (D) 4 (E) 5

..................................................................
... .. ...
... .....
... .......... ...
.. .............. ...
16. What fraction of the regular hexagon in the diagram ..
.
.
..
.
.
.
.
................
................
...
...
...
... .............
. ...
is shaded? ..
..
.
...
.
......................
.
........................
....................
..
...
...
. .
.. . . . . . . ...
..... ............................... .
1 1 3 ...
..
................................
.. . . . . . . . . ... ....
...
(A) (B) (C) ...
... .
.
......................
.
..................... ..
...........................
..
..
...

4 3 8 ...
...
..
.........................................
............................... ..
..
... .............................................. ...
5 1 .
... ...............................
... .................................................. ....
..
.

(D) (E) ... ..................................... ...


......................................................................................................
12 2

17. In a school photo, the 630 pupils are arranged in rows. Each row has 3 more pupils
in it than in the row in front. Of the numbers below, which number of rows is
impossible?

(A) 3 (B) 4 (C) 5 (D) 6 (E) 7

18. A number of runners competed in a race. When Jack finished, there were half as
many runners who had finished before him compared to the number who finished
behind him. Jill was the 10th runner to finish behind Jack and there were twice as
many runners who had finished before her compared to the number who finished
behind her. How many runners were there in the race?
(A) 27 (B) 28 (C) 29 (D) 30 (E) 31
I4

S ....
... ....
. R
....

....
.... ....
.... .....
19. P QRS is a square with side 3. X is ....
....
.... ...
.... ....
.... ....
.... ....
a point on P Q, Y and Z are points on ....
....
....
....
....
....
...

. Z
....
... ....
P S and QR respectively. XY  QS and ....
.... ..
.
.... .. ....
.... .... ...
.... ...... ...
....... ...
...
XZ  P R. The sum of the lengths of ..
........
.... ......
.... .... ......
....
....

.... .... ..
XY and XZ is Y ....
.... ....
....
... .... .......
..
.......
... ......
√ √ √ ...
....
....
.... ......
...
....
....
.
....
..
....
..
. ...
....
....
...
(A) 5 (B) 6 (C) 7 ......
. ....
....
....

.
. .. ... ....
.... ......
. .
.... ....
....
. .... ..
... ....
... .... ... ....
(D) 8 (E) 3 ....
.. .. .
... ...
.......
.. ....
..

P X Q

20. Each of Andrew, Bill, Clair, Daniel and Eva either always lies or is always truthful,
and they know which each of them is.
Andrew says that Bill is a liar.
Bill says that Clair is a liar.
Clair says that Daniel is a liar.
Daniel says that Eva is a liar.
The largest possible number of liars among them can be

(A) 1 (B) 2 (C) 3 (D) 4 (E) 5

Questions 21 to 25, 5 marks each

21. On her birthday in 2007, Rachel’s age is equal to twice the sum of the digits of the
year in which she was born. How many possible years are there in which she could
have been born?

(A) 1 (B) 2 (C) 3 (D) 4 (E) 5

22. A border of black counters is placed around a rectangular ~~~~~~


array of white counters in a way similar to that shown in ~nnnn~
the diagram. If the number of white counters is equal ~nnnn~
to the number of black counters, for how many different ~~~~~~
numbers of white counters can this be done?
(A) 0 (B) 1 (C) 2 (D) 3 (E) 4
I5

23. P QR is a right-angled triangle with


P .............
P R = 3 cm and QR = 4 cm. The ......
......
...... T
.......
square ST UV is inscribed in P QR. .....
..
... ...........
......
......
......
.... ......
......
What is the length, in centimetres, of .
..... ......
.......
...... U
.... .........
.. .. ...........
the side of the square? ...
.
..
.. ......
S .. . .......
. . .......
........ ... ......
...... ... ......
30 12 5 ......
......
...... ...
... .......
......
.......
(A) (B) (C) ......
......
...... ..
.
... ......
......
.......
17 7 3 ......
...... ...
....
.
.. ......
......
......
..
60 60 R V Q
(D) (E)
37 39

24. There are four lifts in a building. Each makes three stops, which do not have to
be on consecutive floors or include the ground floor. For any two floors, there is
at least one lift which stops on both of them. What is the maximum number of
floors that this building can have?

(A) 4 (B) 5 (C) 6 (D) 7 (E) 12

25. A bee can fly or walk only in a straight line between any two corners on the inside
of a cubic box of edge length 1. The bee managed to move so that it visited every
corner of the box without passing through the same point twice in the air or on
the wall of the box. The largest possible length of such a path is
√ √ √ √ √ √ √
(A) 2 + 5 2 (B) 1 + 6 2 (C) 7 2 (D) 3 + 6 2 (E) 4 3 + 3 2

For questions 26 to 30, shade the answer as an integer from 0 to 999 in


the space provided on the answer sheet.
Question 26 is 6 marks, question 27 is 7 marks, question 28 is 8 marks,
question 29 is 9 marks and question 30 is 10 marks.

26. What is the smallest number of odd numbers in the range 1, . . . , 2006 such that,
no matter how these numbers are chosen, there will always be two which add to
2008?

27. Two semicircles of radius 1 are drawn ...............................................


.................... ..... ...........
.......... ..... .... ........
........ ...... .......
...... ...
on the diameter of a semicircle of radius .
.
.........
...
..
.
.
.. ..
..
..
.
.....
.....
....
.. . . ....
2. A circle C touches all three semicir- .
.
...
.......
. ...
.
...
..
..
C .
.
...
...
.
....
....
...
...
..
. . ..
. ....................
...
.. ... ...
cles as shown. If the radius of the circle .
.
.. .
...
.........
...
....
...
... ....
........
..............
...
.. ...
..
...
............
... ..
...........
..
....... ...
.
a . ....... ....
..... .
.
. . ...
....................................... ... ..... ...
... ...... ... . ... ..
C is , where a are b are integers with ... ...
.. ..
...
... ... .... ... ..
.....
.....
b .....
......
...
.. ...
.. ..
.. ..
....
...
....
....
no common factors, then what is the ...
.
....
..
..
..

value of a + b?
I6
28. A lucky number is a positive integer which is 19 times the sum of its digits. How
many different lucky numbers are there?

29. A grid of squares measuring 9 units by 6 units has the two corners removed as
shown:

How many squares of any size are contained within this grid?

30. On my calculator screen the number 2659 can be read upside down as 6592. The
digits that can be read upside down are 0, 1, 2, 5, 6, 8, 9 and are read as 0, 1, 2, 5,
9, 8, 6 respectively. Starting with 1, the fifth number that can be read upside down
is 8 and the fifteenth is 21. What are the last three digits of the 2007th number
that can be read upside down?

***
Intermediate 2007 Answers
Question Answer
1 C
2 C
3 A
4 E
5 C
6 C
7 C
8 B
9 C
10 D
11 A
12 E
13 D
14 D
15 C
16 B
17 D
18 E
19 B
20 C
21 D
22 C
23 D
24 B
25 D
26 503
27 5
28 11
29 142
30 898
A SELECTION OF AUSTRALIAN MATHEMATICS TRUST PUBLICATIONS
Indicate Quantity Required in Box

AUSTRALIAN MATHEMATICS COMPETITION BOOKS


A u s t r a l i a n M a t h e ma t i c s C o m p e t i t i o n
2008 AMC SOLUTIONS AND STATISTICS SECONDARY VERSION – $A35.00 EACH
2008 AMC SOLUTIONS AND STATISTICS PRIMARY AND SECONDARY VERSIONS – $A57.00 FOR BOTH an activity of the australian mathematics trust
Two books are published each year for the Australian Mathematics Competition, a Primary version for the Middle and
Upper Primary divisions and a Secondary version for the Junior, Intermediate and Senior divisions. The books include the
questions, full solutions, prize winners, statistics, information on Australian achievement rates, analyses of the statistics
as well as discrimination and difficulty factors for each question. The 2008 books will be available early 2009.
AUSTRALIAN MATHEMATICS COMPETITION – $A40.00 EACH
T H U R S D AY 31 J U L Y 2 0 0 8
BOOK 1 (1978-1984) BOOK 2 (1985-1991) BOOK 3 (1992-1998)
BOOK 3-CD (1992-1998) BOOK 4 (1999-2005)
These four books contain the questions and solutions from the Australian Mathematics Competition for the years
indicated. They are an excellent training and learning resource with questions grouped into topics and ranked in order INTERMEDIATE DIVISION COMPETITION PAPER
of difficulty.

BOOKS FOR FURTHER DEVELOPMENT OF MATHEMATICAL SKILLS AUSTRALIAN SCHOOL YEARS 9 AND 10
PROBLEMS TO SOLVE IN MIDDLE SCHOOL MATHEMATICS – $A50.OO EACH TIME ALLOWED: 75 MINUTES
This collection of problems is designed for use with students in Years 5 to 8. Each of the 65 problems is presented ready to
be photocopied for classroom use. With each problem there are teacher’s notes and fully worked solutions. Some problems
have extension problems presented with the teacher’s notes. The problems are arranged in topics (Number, Counting,
Space and Number, Space, Measurement, Time, Logic) and are roughly in order of difficulty within each topic.
PROBLEM SOLVING VIA THE AMC – $A40.00 EACH
I N S T RUCT I O N S A N D I N F O RMAT I O N
This book uses nearly 150 problems from past AMC papers to demonstrate strategies and techniques for problem solving. GENERAL
The topics selected include Geometry, Motion and Counting Techniques.
1. Do not open the booklet until told to do so by your teacher.
CHALLENGE! (1991–1998) – $A40.00 EACH
2. NO calculators, slide rules, log tables, maths stencils, mobile phones or other calculating aids are
This book reproduces problems, solutions and extension questions from both Junior (Years 7 and 8) and Intermediate
(Years 9 and 10) versions of the Mathematics Challenge for Young Australians, Challenge Stage. It is a valuable resource permitted. Scribbling paper, graph paper, ruler and compasses are permitted, but are not essential.
book for the classroom and the talented student. 3. Diagrams are NOT drawn to scale. They are intended only as aids.
The above prices are current to 31 December 2008. 4. There are 25 multiple-choice questions, each with 5 possible answers given and 5 questions that
On line ordering and details of other AMT publications are available on the Australian Mathematics Trust’s web require a whole number between 0 and 999. The questions generally get harder as you work
site www.amt.edu.au
through the paper. There is no penalty for an incorrect response.
5. This is a competition not a test; do not expect to answer all questions. You are only competing against
PAYMENT DETAILS Payment must accompany orders. Please allow up to 14 days for delivery. your own year in your own State or Region so different years doing the same paper are not compared.
Please forward publications to: (print clearly) 6. Read the instructions on the Answer Sheet carefully. Ensure your name, school name and school
Name: year are filled in. It is your responsibility that the Answer Sheet is correctly coded.
Address: 7. When your teacher gives the signal, begin working on the problems.

Country: Postcode:
THE ANSWER SHEET
1. Use only lead pencil.
POSTAGE AND HANDLING - within Australia, add $A3.00 for the first book and $A1.00 for each additional book
- outside Australia, add $A13.00 for the first book and $A5.00 for each additional book 2. Record your answers on the reverse of the Answer Sheet (not on the question paper) by FULLY
Cheque/Bankdraft enclosed for the amount of $A colouring the circle matching your answer.
Please charge my Credit Card (Visa, Mastercard) Amount authorised:$A Date: / / 3. Your Answer Sheet will be read by a machine. The machine will see all markings even if they are
Cardholder’s Name (as shown on card):
in the wrong places, so please be careful not to doodle or write anything extra on the Answer Sheet.
If you want to change an answer or remove any marks, use a plastic eraser and be sure to remove
Cardholder’s Signature: Tel (bh):
all marks and smudges.
Card Number: Expiry Date: /
INTEGRITY OF THE COMPETITION
All payments (cheques/bankdrafts, etc) should be in Australian currency
and be made payable to AUSTRALIAN MATHEMATICS TRUST and sent to: The AMC reserves the right to re-examine students before deciding whether to grant official status
Australian Mathematics Trust, University of Canberra ACT 2601, Australia. to their score.
Tel: 02 6201 5137 Fax: 02 6201 5052

©AMT Publishing 2008 amtt limited acn 083 950 341


Intermediate Division

Questions 1 to 10, 3 marks each

1. The value of 802 − 208 is

(A) 606 (B) 604 (C) 504 (D) 694 (E) 594

2. Given that 1.08 × 1.8 = 1.944, the value of 108 × 18 is

(A) 194.4 (B) 1944 (C) 19.44 (D) 19 440 (E) 19 400

..
...
...
....
3. In the diagram, the sides of the triangles are ...
..
...
.
107◦
........
... ......
extended and three angles are as shown. The ...
..
...
.
. ....
...
....
... ....
value of x is ..
.
.
.
..
.... ....
....
....
....
..
.... ....
... ....
. ....
(A) 100 (B) 110 (C) 120 .
.
..
.
.
..
... ....
....
....
.
.. ....
.. ....
(D) 130 (E) 140 153◦
.
..
.
..
.
.
..
... ....
....
....
....

x◦
....
....
...

200 × 8
4. The value of is
200 ÷ 8
(A) 1 (B) 8 (C) 16 (D) 64 (E) 200

5. The digits 5, 6, 7, 8 and 9 can be arranged to form even five-digit numbers. The
tens digit in the largest of these numbers is

(A) 5 (B) 6 (C) 7 (D) 8 (E) 9

6. Four consecutive odd numbers add up to 48. What is the largest of these numbers?

(A) 13 (B) 15 (C) 17 (D) 19 (E) 21


I2
7. A rectangle has an area of 72 square centimetres and the length is twice the width.
The perimeter, in centimetres, of the rectangle is

(A) 34 (B) 36 (C) 42 (D) 48 (E) 54

8. What percentage of y is x?
y x x 100y 100x
(A) (B) (C) (D) (E)
x 100 y x y

L ..............
... .....
...
S
.... Q
9. In the diagram, triangles P QR and LMN are ...
..
.....
....
.
.................
...............
.................
............... ....

20◦
... .
............... ....... ..
. .
.. ..
both equilateral and  QSM = 20◦ . What is the . ..........
.......... ..... ..
.................. ....
P ...
...
...
..
.....
....
.... ..
.... ..
..
.
..

value of x?
... .
... .... ......
... .. .. ....
... .. .. .......
... ... ... ....
M
..... . ....
..... ... ...
....... . .....
..
(A) 70 (B) 80 (C) 90 .... .....
... ....
...
... .............
... ..
.. ........
... ... .. ......
.. .......
x◦
...
... ... ........
(D) 100 (E) 110 ...
...
...
...
...
..
.
.........
... ............ ....
.. .
.
. . .
... .............. ..... ...
N .............. ... ...
.....

10. When 10002008 is written as a numeral, the number of digits written is

(A) 2009 (B) 6024 (C) 6025 (D) 8032 (E) 2012

Questions 11 to 20, 4 marks each


.......................................
........... ........
....... .....
..... ....
... .... ....
11. Anne designs the dart board shown, where she .....
.
.
....
. ........ ...... ..................... .............
......
...
...
...
...
. .. ...
....
scores P points in the centre circle, Q points in .. .... ..
. . .... ..
... ... ..
.... .... ............................ ..
. ..
...
... ... ... ..
the next ring and R points in the outer ring. She
.. . .. . ..
... .. .. . ... ...
....
P ..
... ... . .
.
. ..
... ... .. ..
. ...
. ..
... .. .... .. .. ...
.
throws three darts in each turn. In her first turn, ..
...
..
...
..
...
....
..................... ..... .
...
..
. .
.
..
..
.
... ... ..
Q
..
she gets two darts in ring Q and one in ring R and ...
...
...
....
....
... .........
................................
.
... ....
...
....
.
..
.... ...
.... ....
scores 10 points. In her second turn, she gets two .....
R
.....
.....
........ ..........
............ ....
...................................
in circle P and one in ring R and scores 22 points.
In her next turn, she gets one dart in each of the regions. How many points does
she score?

(A) 12 (B) 13 (C) 15 (D) 16 (E) 18


I3
12. How many different positive numbers are equal to the product of two odd one-digit
numbers?

(A) 25 (B) 15 (C) 14 (D) 13 (E) 11

13. Points A, B, C, D and E are nodes of a square grid as shown. Which of these five
points forms an isosceles triangle with the other two vertices at X and Y ?
E D C
... ... ...
• • •
.................................................................................................................
... ... ...
.......................................................................................................................
X• ...
....
..
....
...
...
•B ..
...
....
..
..
....
.. .. ..
..
....
.
.... ....
•A
.................................................................................................................
.... .....
.
....
... .... .... ... .. ..
...............................................................................................................
... .
. .
. .
. ..
. ...
..... .... .... ..... ... ..
.................................................................................................................
... ... ... ... ... ...
.... ... ... ... ... ..
.................................................................................................................

Y

(A) A (B) B (C) C (D) D (E) E

14. A Fibonacci die has the numbers 1, 1, 2, 3, 5 and 8 on it. Two such dice are
thrown. What is the probability that the number on one die is larger than the
number on the other?
1 5 2 5 7
(A) (B) (C) (D) (E)
2 9 3 6 9

15. A fishtank with base 100 cm by 200 cm and depth 100 cm contains water to a depth
of 50 cm. A solid metal rectangular prism with dimensions 80 cm by 100 cm by
60 cm is then submerged in the tank with an 80 cm by 100 cm face on the bottom.
.... ...
.... ....
6 ....
...
....
....
...
....
...
....
... ....
... ....
.... ....
.... ....
. .

......... . . . . . . . . . . . . . . . . . . . . . . . . . . . . ....
........ .
................................... ....................................................................................... .... ....
100 ...................... ...........................................................
. . . .... . . . . . . . . . . ............. .
................................. ................................................................................................
....
....
....
....
....
....
....................... ............................................................. .... ....
...................... ...............................................................
. . . . . . . .... . . ................. .
.................................. ............................................................................................................
...........................................................................................
................... ................... . . . ........................................
................... ......................................................................
? ...................... ..........................................................................................................
. . . . . . . . . . . . .
.................... ................................ . . . . ...........................
.................. ....................................................................
............... .........................................................................
.............. ...................... . . . . . . . . . ...........................
............ ........................................................................
50 ....
60
.......... ......................................................................... ....
100 ........ .........................................................................
...... ..........................................................................
.......................................................................... 80
...
....
...
..

200 100

The depth of water, in centimetres, above the prism is then

(A) 12 (B) 14 (C) 16 (D) 18 (E) 20


I4

16. What is the smallest whole number which gives a square number when multiplied
by 2008?

(A) 2 (B) 4 (C) 251 (D) 502 (E) 2008

17. The interior of a drinking glass is a cylinder of diameter 8 cm and height 12 cm.
The glass is held at an angle of 45◦ from the vertical and filled until the base is
just covered. How much water, in millilitres, is in the glass?

(A) 48π (B) 64π (C) 96π (D) 192π (E) 256π

18. A number is less than 2008. It is odd, it leaves a remainder of 2 when divided by
3 and a remainder of 4 when divided by 5. What is the sum of the digits of the
largest such number?

(A) 26 (B) 25 (C) 24 (D) 23 (E) 22

Q
.........................
............... ............... ........................
.........
19. P R and QS are perpendicular diameters .......
......
.......
. ...
.......... ..
.................
.................................
........
.....
.....
....
.. ....................................... ....
drawn on a circle centre O. The points ... ...
.
.... .................................................
.
...
....
... ...
....................... . . . . . ... ...
. . . .
.............................................................. . . . . .

T , U, V and W are the midpoints of P O, ..


.
..
.
...
.
..
...
. ..
U
..........................................................................
...................................... .................................
...
...
..
..
... .................................. ................................
. . . . .
.... . . . . . . . .......... . ..
.. .. ................................... . . . . . . . ..
QO, RO and SO respectively.
. .
................................. ................. . ..... ... . . . ..
... ...
.................................... .
. . . . . . .
.............................. . . . .. ..
..
... ..
. ... ... . . . . . . . . . . . ..
... ............................................. .............................
......................... .... ...
. . .
The fraction of the circle covered by the P
... ..............................
.........................................................
................................. O ..
V.
.. . . .
.
.
.
.
.
.
.
............................ ...
.
.
.
.
.
............................. ......
.
. . .. R
........... ....................
shaded area is ...
T
... ............................................
... ................................
................................ .................................................... ....
. . . .. . .
.............................. ....
.
.. .................................. ............ . . ............ ..
... ................................... ............................. ..
.............................. ............ . . .............
.. ............................................. ..
1 1 3 ..
..
..............................
............................... ................................
.............................. .............................
. .
....
(A) (B) (C) ...
... .....................................................
............................................................ ...
..
.

2π π 2π ...
...
...
...
W
...................................................
............................................
.................................... ...
.....
...
.... ............................... ...
2 5 ....
.....
.....
............................
...........................
.....
....
....
(D) (E) ........
..........
.................
... . .
............. .................... .....................
.... .
... . .

π 2π ..............................

20. Three numbers p, q and r are all prime numbers less than 50 with the property
that p + q = r. How many values of r are possible?

(A) 0 (B) 2 (C) 4 (D) 6 (E) 8


I5

Questions 21 to 25, 5 marks each

21. Farmer Taylor of Burra has two tanks. Water from the roof of his farmhouse is
collected in a 100 kL tank and water from the roof of his barn is collected in a
25 kL tank. The collecting area of his farmhouse roof is 200 square metres while
that of his barn is 80 square metres. Currently, there are 35 kL in the farmhouse
tank and 13 kL in the barn tank.
Rain is forecast and he wants to collect as much water as possible. He should:
(A) empty the barn tank into the farmhouse tank
(B) fill the barn tank from the farmhouse tank
(C) pump 10 kL from the farmhouse tank into the barn tank
(D) pump 10 kL from the barn tank into the farmhouse tank
(E) do nothing

22. If the tens digit of a perfect square is 7, how many possible values can its units
digit have?

(A) 0 (B) 1 (C) 2 (D) 3 (E) 4

23. Twenty-five different positive integers add to 2008. What is the largest value that
the least of them can have?

(A) 65 (B) 66 (C) 67 (D) 68 (E) 69

24. P QR is an equilateral triangle. The Q ....


... ...
point U is the mid-point of P R. Points ...
...
... .....
...
...
... ...
T and S divide QP and QR in the ra- .
..
.. ... ...
...
...
T S
.. ...
.. ..
tio 1 : 2. The point of intersection of .
........
.. .......
.. ..
.
.....
.... .....
... ..... .
... ...
P S, RT and QU is X. If the area of
.... .... ...
... .....
.... ........ ...

..
....
..
. ..
.......
... .......
.. X ...
...
QSX is 1 square unit, what is the . ... ...
... ...... .....
. ..... ...
.
.. ...... ...... ...
... .....
. ...
...
... .... ...
area, in square units, of P QR? ..
.
..
.. ...
............. .. ...
.....
......
.......
...
...
...
... ......
. .. ...
... .....
.... .. . ..
....
...
... ...... ..... ....
. .
(A) 6 (B) 8 (C) 9 .
.... ......
.. ......
..
..... ....
..... ..
......... .... ..
........
...... ..
.....................................................................................................................................................................................................
(D) 12 (E) 18
P U R

25. A two-digit number n has the property that the sum of the digits of n is the same
as the sum of the digits of 6n. How many such numbers are there?

(A) 0 (B) 3 (C) 4 (D) 8 (E) 10


I6

For questions 26 to 30, shade the answer as an integer from 0 to 999 in


the space provided on the answer sheet.
Question 26 is 6 marks, question 27 is 7 marks, question 28 is 8 marks,
question 29 is 9 marks and question 30 is 10 marks.

.........
.........
......... ....
S
......... ...
26. In the diagram, .........
.
.... .
. .
. ..
..

 OP Q =  OQR =  ORS = 90◦ .


R .........
........
.. .
............. .
.
.
..
..

........... ...
... ......... ...
.... ... ..
OP = 4 cm, P Q = 3 cm and QR = 12 cm. .. .
.. .. ...
12 ....
...
... ..
..
..
..
..
... .. ...
The perimeter of the pentagon OP QRS is ....
....
.
. ..
..
..
.
.
..
...
.

Q
..
.... .. ..
. ... .. ..
.
188 cm. ......... ......
.... .........
..
.. ...
..
.. ...
.
.
..
..... .. ..
... ..
What is the area, in square centimetres, of ...
...
.....
.....
.....
.....
..
...
..
..
.
..
..
.
... ..
the pentagon OP QRS?
..... .. ..
... ..... .. ..
3 ...
...
...
.....
.....
.....
..
..
..
.
.
.
...
..
.....
.... ..... .... ....
.......... ..... .. ..
..... ... ..
......................................................................................................

P 4 O

27. A rectangular prism 6 cm by 3 cm by 3 cm is made up by stacking 1 cm by 1 cm


by 1 cm cubes. How many rectangular prisms, including cubes, are there whose
vertices are vertices of the cubes, and whose edges are parallel to the edges of the
original rectangular prism? (Rectangular prisms with the same dimensions but in
different positions are different.)

28. The number 2008! (factorial 2008) means the product of all the integers 1, 2, 3, 4,
. . . , 2007, 2008. With how many zeroes does 2008! end?

29. Let us call a sum of integers cool if the first and last terms are 1 and each term
differs from its neighbours by at most 1. For example, the sum 1 + 2 + 3 + 4 + 3 +
2 + 3 + 3 + 3 + 2 + 3 + 3 + 2 + 1 is cool.
How many terms does it take to write 2008 as a cool sum if we use no more terms
than necessary?

30. All the vertices of a 15-gon, not necessarily regular, lie on the circumference of a
circle and the centre of this circle is inside the 15-gon. What is the largest possible
number of obtuse-angled triangles where the vertices of each triangle are vertices
of the 15-gon?

***
Intermediate 2008 Answers
Question Answer
1 E
2 B
3 A
4 D
5 A
6 B
7 B
8 E
9 B
10 C
11 D
12 C
13 B
14 E
15 B
16 D
17 B
18 A
19 C
20 D
21 D
22 B
23 D
24 D
25 D
26 582
27 756
28 500
29 89
30 445
A u s t r a l i a n M a t h e ma t i c s C o m p e t i t i o n
an activity of the australian mathematics trust

t h u r s d ay 6 A u g u s t 2 0 0 9

intermediate Division Competition Paper


australian School Years 9 and 10
time allowed: 75 minutes

Instr uct ions a nd I nf or m ati on


GENERAL
1. Do not open the booklet until told to do so by your teacher.
2. NO calculators, slide rules, log tables, maths stencils, mobile phones or other calculating aids are
permitted. Scribbling paper, graph paper, ruler and compasses are permitted, but are not essential.
3. Diagrams are NOT drawn to scale. They are intended only as aids.
4. There are 25 multiple-choice questions, each with 5 possible answers given and 5 questions that
require a whole number answer between 0 and 999. The questions generally get harder as you work
through the paper. There is no penalty for an incorrect response.
5. This is a competition not a test; do not expect to answer all questions. You are only competing against
your own year in your own State or Region so different years doing the same paper are not compared.
6. Read the instructions on the Answer Sheet carefully. Ensure your name, school name and school
year are filled in. It is your responsibility that the Answer Sheet is correctly coded.
7. When your teacher gives the signal, begin working on the problems.

THE ANSWER SHEET


1. Use only lead pencil.
2. Record your answers on the reverse of the Answer Sheet (not on the question paper) by FULLY
colouring the circle matching your answer.
3. Your Answer Sheet will be read by a machine. The machine will see all markings even if they are
in the wrong places, so please be careful not to doodle or write anything extra on the Answer Sheet.
If you want to change an answer or remove any marks, use a plastic eraser and be sure to remove
all marks and smudges.

INTEGRITY OF THE COMPETITION


The AMC reserves the right to re-examine students before deciding whether to grant official status
to their score.
Intermediate Division

Questions 1 to 10, 3 marks each

1. (2000 + 9) + (2000 − 9) equals

(A) 4000 (B) 4009 (C) 200 (D) 2000 (E) 5000

.
...
... ...
.... ...
.... ...
...
... ...
... ...
....
... ...
.... ....

2. In the diagram, x equals


.......
.....
... ...
x◦
.. ......
.
.
. ...
.. ....
... ....
... ...
....
..
(A) 140 (B) 122 (C) 80 ...
.
.
.. ....
....
...
... ...
....
..
(D) 90 (E) 98 .
..... ...
.
.....................................................................................................................................................................................
....
..
...
...
...
.
140◦ 122◦ ...
....
..
.
...
...

1 2 3 4
3. The value of × × × is
2 3 4 5
1 5 1 1 1
(A) (B) (C) (D) (E)
5 7 6 15 60

4. Which of the following has the largest value?


1 1 1 1 1 1 1 1 1
(A) (B) + (C) × (D) − (E) ÷
3 3 3 3 3 3 3 3 3

5. Which of the following values can replace the box so that

0.1 × 0.2 × 0.3 × 0.4 × = 0.12 ?

(A) 500 (B) 50 (C) 5 (D) 0.5 (E) 0.05

6. If 3k = 930 then k equals

(A) 15 (B) 30 (C) 40 (D) 60 (E) 90


I2
7. (x − y) − 2(y − z) + 3(z − x) equals

(A) −2x − 3y + 5z (B) −2x − 3y − z (C) 4x + y − z


(D) 4x + 3y − z (E) 2x + 3y − 5z

8. The point (k, 17) lies on the line joining the points (1, 5) and (4, 11). The value of
k is

(A) 37 (B) 14 (C) 8 (D) 6 (E) 7

9. Paperback books cost $5 each and hardcover books cost $7 each. I spend exactly
$86 on books of these two types. What is the maximum number of books that I
could have bought?

(A) 10 (B) 14 (C) 16 (D) 18 (E) 20

10. The figure P QRS is a rectangle


divided into 10 squares as shown. P Q
The perimeter of this rectangle
is 21 centimetres. In centime-
tres, what is the perimeter of each
square ?
(A) 2.1 (B) 3 (C) 6 S R
(D) 8.4 (E) 12

Questions 11 to 20, 4 marks each

11. In a school of 1000 students, 570 are girls. One-quarter of the students travel to
school by bus and 313 boys do not go by bus. How many girls travel to school by
bus?

(A) 7 (B) 63 (C) 153 (D) 180 (E) 133


I3
P ..
..
...
...
. ........
......... ..
Q
......... ....
12. In the diagram, triangles P QT ,
...
...
..
...
2x◦ ..... .
...........
.........
....
....... ...
... .. ................
.. ..... ...
.......... ..
QT S and QRS are isosceles and ...
...
... ..
.........
....
............ .
.
...
....

� P QR is a right angle. Angles P QT


..
....................
....
.........
...
..
.. 2x◦
T
...
.... 5x◦
.... .... ...
.
......
..

and RQS are 2x◦ and angle QT S is ...


...
....
.... ....
.. . ...........
... ...
5x◦ . The value of x is ...
...
...
.... ...
....
..

....
... ...
... ...
..
(A) 10 (B) 12 (C) 14 ....
... ...
.... ..
.....
.

...........
...........
(D) 15 (E) 20 S ...........
...........
...........
...........
........
R

13. This pattern consists of squares. Areas,


in square units, of four of the squares
are shown. Given that X and Y are X
also squares, their areas are respec- Y
tively
1 1
(A) 16, 25 (B) 16, 36 (C) 25, 36 9
4
(D) 25, 64 (E) 25, 100

14. Two numbers P and Q are such that P is 40% greater than Q. The ratio P : Q is

(A) 40 : 1 (B) 5 : 7 (C) 5 : 3 (D) 5 : 2 (E) 7 : 5

15. What is the last digit of 6 × 82009?

(A) 0 (B) 2 (C) 4 (D) 6 (E) 8

16. Two dice are each numbered from 1 to 6, but are biased so that each is twice as
likely to land on any of the even numbers as on any of the odd numbers. The two
dice are rolled and the numbers multiplied together. What is the probability that
the resulting product is odd?
1 2 1 4 2
(A) (B) (C) (D) (E)
9 9 3 9 3

17. An eyebrow is an arrangement of the numbers 1, 2, 3, 4 and 5 such that the second
and fourth numbers are each bigger than both their immediate neighbours. For
example, (1, 3, 2, 5, 4) is an eyebrow and (1, 3, 4, 5, 2) is not.
The number of eyebrows is

(A) 16 (B) 12 (C) 15 (D) 24 (E) 18


I4

18. P QRS is a rectangle. X is halfway along


P Q, Y is a third of the way along QR S..... ...
...
Z R
... ...
....
.... ...
...
and Z is a quarter of the way along RS. ....
....
....
....
...
...
...
.... ...
What fraction of the area of P QRS is ....
....
....
...
...
...
...
.... ...
represented by the quadrilateral XY ZS? ....
....
....
....
...
...
...
..

1 7 2
...
....
.... .
...
...
...........
...
............ Y
....
.... .
...
............
(A) (B) (C) ....
....
......
.........
..........
.... ...................
2 12 3 .....

3 3 P X Q
(D) (E)
4 5

9
19. The difference between a positive fraction and its reciprocal is . The sum of the
20
fraction and its reciprocal is
41 20 25 41
(A) (B) (C) (D) (E) 5
40 9 16 20

20. A circular bottle with dimensions, in centimetres, is shown partially filled with
water (figure 1).
..
................................ ...........................................
............ .........
1 ........ .....
............... .
...... ...
........................................... .
................................................................................................................................
......................... .... ........ .
.......
....
... � ...
...
... ............
... .............. �
......................................................................
...
. .
......... ....
...
... ... ... ....
... ... ... ...
... ... ... ...
...
...
...
3 ....
..
...
...
...
...
h ...
.....
.
............... ...... . ...
... .
............... .
.... ........................... .... . .......... ...... .
.. ....
. .
..
. . ............
...
...
...
............ . ..
. .... ....
. .. ... ........... .. ........................... .... ... ... .... ... ... ........................... .....
........ . .......
... ............ ... ... .... ... ... .... ... ... .... ... .. ...........
�2 �
.. . . . .....
.. ... .... .
.
. . ............. ... ... .... ... ... .... ... ... .... ... ... .........
... ..... ....................................................
.
. ....................................................................................................................................................
...... ....................... ... ................... ... ... .... ... ... .... ... ... .... ... ... ............
... ...... ........
... ............ . ..
.. ...
. . ..... ................. ... ... .... ... ... .... ... ... .... .......................
................. .......... ... ...................................... ... ... .... ................................ ... ......
...
... ........................................................... ..
... .... ....... ... ... ............................................... .... ... ... .......
...
... ...
. . .
.
......... . . . . . . . . ..........
... ......................... .... ... ... .... ... ... ... ....................... ....
..
....
. ..
.. ..
.. .
.. .
.. .....
.. ...
......
...
.. 2 .... ....... ... ... .... ... ... .... ... ... .... ... ... .....
.... ....... ... ... .... ... ... .... ... ... .... ... ... ......
.... ....... ... ... .... ... ... .... ... ... .... ... ... .......
... ........ .. ... ... .... ... ... .... ... ... .... ... ... . ......... ... ..... ....................................................................................................... ... ......
....... .... ... ... .... ... ... .... ... ... .... ... ... ... ........ ... ................... . ... .... ... ... .... ... ... .... ................. ......
....... .... ... ... .... ... ... .... ... ... .... ... ... ... ....... .............. .... ... ... .... ... ... .... ... ... .... ... .. ..............
.............. ... ... .... ... ... .... ... ... .... ... ... ............ ....... ....... ... ... .... ... ... .... ... ... .... ... ... ........
.. ................... ... .... ... ... .... ... ... .... ... ................. ..... .... ....... ... ... .... ... ... .... ... ... .... ... ... .....
..... . . . . . . . . . . . . . . . . . . . . . .....
... .. .... ...................... . ... ... .... ... ... ..................... ... .... ........ ... ... ... .... ... ... .... ... ... .... ... ........
... .. .... ... ... .................................................................. ... ... ... ..... .......... . . . . . . . . . . . . . . . . ...........
... .. .... ... ... .... ... ... .... ... ... .... ... ... ... ...... ................ ... ... ... .... ... . ................
.....................................................
... .. .... ... ... .... ... ... .... ... ... .... ... ... ... .... .. . . . . ...
.... .. .... ... ... .... ... ... .... ... ... .... ... ... ... ......
... .. .... ... ... .... ... ... .... ... ... .... ... ... ... ....
.. .. .... ... ... .... ... ... .... ... ... .... ... ... ... .....
4 ... .. ... .......
... .. ... .......
... .. ... ......
.... .. .... ... ... .... ... ... .... ... ... .... ... ... ... ..... ... .. ... .......
... .. .... ... ... .... ... ... .... ... ... .... ... ... ... ...... ... .. ... ........
... .. .... ... .......................................................................... .. ... ... .... ... .. ... .......
... .. .......................... .... ... ... .... ... ... .............................. ..... ... .. ... .......
... .............. ... ... .... ... ... .... ... ... .... ... ... ................. ... .. ... ........
...... .... ... ... .... ... ... .... ... ... .... ... ... ... ...... .... .. ... .......
...................................................................................................................................................... ...... ... ........
.... . . . . . . . . . . . . . . . . . . . . . .... ...............
...... .. ... ... .... ... ... .... ... ... .... ... ... . ......
5
.......... . . . . . . . . . . . . . . . . ..........
............... . . . . . . . . . . . .............
.....................................................................

figure 1 figure 2
The bottle is sealed and then turned upside down (figure 2). The height h, in
centimetres, of the air in the upturned bottle is
1 7 1 3
(A) 2 (B) 2 (C) 2 (D) 2 (E) 2
3 25 5 25
I5

Questions 21 to 25, 5 marks each

21. I bought a map of Australia, unfolded it and marked eight places I wanted to visit.

I then refolded the map and placed it back on the table as it was. In what order
are my marks stacked from top to bottom?

(A) RTYQKAWP (B) YKRAWTPQ (C) RTQYKAWP


(D) YKTPRAWQ (E) YKWARTPQ

22. A palindromic number is a ‘symmetrical’ number which reads the same forwards
as backwards. For example, 55, 101 and 8668 are palindromic numbers.
There are 90 four-digit palindromic numbers.
How many of these four-digit palindromic numbers are divisible by 7?

(A) 7 (B) 9 (C) 14 (D) 18 (E) 21

P ......................................................................................U
..............................................................................................
Q
23. P QRS is a square. T and U are mid- ... ....
... .... ..
. .. . ......
........
..
...
... ....
... ....... ...
.... ........
points of the sides P S and P Q respec- ..
...
...
...
.
....
...
.... .
..
.
..
.
....
........
. ......
. .
....
.. ...
...
...
.... .. . .
.....
... .. .
.... ...
tively. T Q and SU intersect at V .What ...
...
...
....
.. ... ...........
... .. .......
...........
. ....
...
...
....
. .. ...
... ..... ... .....
. ...
fraction of the area of the square is the ...
...
... ....... ..
...
........
... ..
..
..... ..
.
..
.
....
....
..
...
....
area of quadrilateral QV SR? T .........
... V .
..
.
.
.
.. ....
.....
.
....
..
...
.... . .... ...
... ... .... ...
... ... .... ...
....
1 5 2 ...
...
...
...
.
. ....
.... .
.
.
...

(A) (B) (C) ...


....
.
...
..
.. ......
....
.
.
.
.
.
....
2 8 3 ...
.. ....
..
....
....
.... ...
...
... .. ....
3 5 ... ..
... ...
....
.... ....
..

(D) (E) ... ..


..............................................................................................................................................................
.... ..

4 9 S R
I6

24. In 3009, King Warren of Australia suspects the Earls of Akaroa, Bairnsdale, Clare-
mont, Darlinghurst, Erina and Frankston are plotting a conspiracy against him.
He questions each in private and they tell him:
Akaroa: Frankston is loyal but Erina is a traitor.
Bairnsdale: Akaroa is loyal.
Claremont: Frankston is loyal but Bairnsdale is a traitor.
Darlinghurst: Claremont is loyal but Bairnsdale is a traitor.
Erina: Darlinghurst is a traitor.
Frankston: Akaroa is loyal.
Each traitor knows who the other traitors are, but will always give false informa-
tion, accusing loyalists of being traitors and vice versa. Each loyalist tells the truth
as he knows it, so his information on traitors can be trusted, but he may be wrong
about those he claims to be loyal.
How many traitors are there?

(A) 1 (B) 2 (C) 3 (D) 4 (E) 5

25. Which of the following cannot be the last digit of the sum of the squares of seven
consecutive numbers?

(A) 3 (B) 5 (C) 6 (D) 7 (E) 8

For questions 26 to 30, shade the answer as an integer from 0 to 999 in


the space provided on the answer sheet.
Question 26 is 6 marks, question 27 is 7 marks, question 28 is 8 marks,
question 29 is 9 marks and question 30 is 10 marks.

26. What is the smallest positive integer which, when divided by each of 2, 3, 4, 5, 6
and 7, will give in each case a remainder that is one less than the divisor?

27. We say a number is ascending if its digits are strictly increasing. For example, 189
and 3468 are ascending while 142 and 466 are not. For which ascending 3-digit
number n (between 100 and 999) is 6n also ascending?

28. A magician deposits the same number of rabbits (at least one) at each of five
houses. To get to the first house he crosses a magic river once, and to get to any
house from another, he also crosses a magic river once. Each time he crosses a
magic river, the number of rabbits he has doubles. He has no rabbits left when he
leaves the fifth house. What is the minimum number of rabbits he could have at
the start?
I7

29. Consider this sequence of patterns made from hexagons.


............................
... ...
.. ...
..
. .... ...
...
............................
... ... ................................ ..
... ... .... ...
....
.. ... .... ... ..
. ... .. .. . ...
. . .
.
.
...
............................ ................................ .. ............................... ..............................
..
...
..
. ... .. .. ... ... ...
..
...
. . ... .... ... ... ... ...
...
...
... ... ... .. ... ... ... ...
..... ..... .................. ..
.
.......... ..... ................................ ..
... ... ... .. ... ... . ... ...
.. ... .. ... ..
. ... ..
...
..
. ... ... ... ... ..
. ... ....
...
............................... ... ... ...
... . ... ..
.............................. ... ............................. .
.............................
... ... ... ... ...
... ... ... ... ...
...
... ... ... ... ...
. ... ...
........................... ............................. .
...
... .....
... ...
... .
.............................

pattern 1 pattern 2 pattern 3


The first pattern consists of 6 line segments, the second of 15 and the third of 27.
How many line segments are in the 11th pattern?

30. The country of Big Wally has a railway which runs in a loop 1080 km long. Three
companies, A, B and C run trains on the track and plan to build stations. Company
A will build three stations, equally spaced at 360 km intervals. Company B will
build four stations at 270 km intervals and Company C will build five stations at
216 km intervals.
C
B ..............................•
...................
..........

•A
........
...
.....

.. ..........
..........
......
A C ................. .....
....
....
......

......... ......
.•
. ...
.... C
.
............... ..
.................... .....
..
• ....
...
...
.
.... ...
...
. ...
. .. ...
B• .
....
...
..
..
..
•B
...
...
.
...
... ...
..
... ...
... ...
....
.... .
......
..... .....
................. ...........

................ .................
• ..........
........ ...........
..........
. C
C
....
....
.... ....
.....
.
....
..
..

.....
...... ....
.......
B
..........

..............
...
...
..

A
The government tells them to space their stations so that the longest distance
between consecutive stations is as small as possible. What is this distance in
kilometres?

***
Intermediate 2009 Answers
Question Answer
1 A
2 E
3 A
4 E
5 B
6 D
7 A
8 E
9 C
10 C
11 E
12 D
13 D
14 E
15 E
16 A
17 A
18 B
19 D
20 E
21 E
22 D
23 C
24 D
25 D
26 419
27 578
28 31
29 231
30 174
A u s t r a l i a n M a t h e ma t i c s C o m p e t i t i o n
an activity of the australian mathematics trust

T H U R S D AY 5 A U G U S T 2 01 0

INTERMEDIATE DIVISION COMPETITION PAPER


AUSTRALIAN SCHOOL YEARS 9 AND 10
TIME ALLOWED: 75 MINUTES

INSTR UC T IONS A ND I NF OR M ATI ON


GENERAL
1. Do not open the booklet until told to do so by your teacher.
2. NO calculators, slide rules, log tables, maths stencils, mobile phones or other calculating aids are
permitted. Scribbling paper, graph paper, ruler and compasses are permitted, but are not essential.
3. Diagrams are NOT drawn to scale. They are intended only as aids.
4. There are 25 multiple-choice questions, each with 5 possible answers given and 5 questions that
require a whole number answer between 0 and 999. The questions generally get harder as you work
through the paper. There is no penalty for an incorrect response.
5. This is a competition not a test; do not expect to answer all questions. You are only competing against
your own year in your own State or Region so different years doing the same paper are not compared.
6. Read the instructions on the Answer Sheet carefully. Ensure your name, school name and school
year are filled in. It is your responsibility that the Answer Sheet is correctly coded.
7. When your teacher gives the signal, begin working on the problems.

THE ANSWER SHEET


1. Use only lead pencil.
2. Record your answers on the reverse of the Answer Sheet (not on the question paper) by FULLY
colouring the circle matching your answer.
3. Your Answer Sheet will be read by a machine. The machine will see all markings even if they are
in the wrong places, so please be careful not to doodle or write anything extra on the Answer Sheet.
If you want to change an answer or remove any marks, use a plastic eraser and be sure to remove
all marks and smudges.

INTEGRITY OF THE COMPETITION


The AMC reserves the right to re-examine students before deciding whether to grant official status
to their score.

©AMT Publishing 2010 amtt limited acn 083 950 341


Intermediate Division

Questions 1 to 10, 3 marks each

3×4
1. The value of is
6
1 3
(A) (B) 1 (C) (D) 2 (E) 3
2 2

..
...
2. In the diagram, the value of x is ..
...
...
...
...
...
...
...
(A) 15 (B) 40 (C) 55 ...
...
..............
..............
..............
..............
x◦ ...
..
.............. ...............
(D) 75 (E) 80 ......................................................................................
......
......
150◦
......
45◦
......
......
......
......
......
......
......
......
......
......
......
......

3. The value of 2010 − 20.10 is

(A) 1990.09 (B) 1990.9 (C) 1989.09 (D) 1989.9 (E) 1998.9

3 m
4. If m = 3 and n = − , then equals
5 n
9 5
(A) −5 (B) 5 (C) − (D) − (E) 15
5 3

5. In the diagram,
......
... ...
... .....
....
. ...
. ...
... ...

.....
...
... 80◦ ...
...
...
...
... ...
.. ...
.... ...
.... ...
.... ...
...
..
.
.
..
...
.
.
40◦ ...
...
...
...................................................................................................................................................................................................................................................................
.
... ..
... ......
...
...
...
...
50◦ ..
......
......
.
......
......
...
... ......
... ......
... ......
... ......
...
... .
..........
.
......
...
...
... x◦
... ............
......
......
...

the value of x is

(A) 50 (B) 60 (C) 70 (D) 80 (E) 90


I2

6. Consider all the integers from 1 to 100 inclusive. What is the difference between
the sum of all the even numbers and the sum of all the odd numbers?

(A) 0 (B) 25 (C) 50 (D) 100 (E) 200

7. Paul’s three children have birthdays in the same ...........


...
............................................
........
.. ............ .
.... .......
. ......
week. He bought a circular birthday cake and ...... ......
......... .
. .....
.............. .
. .....
....
... ...... ...
divided the cake in proportion to their ages as ....
...
.
. .....
.......
.
......
...
.
...
...
...
..
. ....... .
. .
.... ... .........
shown. If none of his children is older than 17, ...
...
.....
.....
.....
..... .... .... .......... ...... ....... . .....
...
...
........ ...
what is the sum of their three ages? ..
... 45◦ .....
..... . .................
. . ...
....... .......... ....... ....... ....... ....... ....... .................. ....... ....... ....... ....... ....... .........
30◦ ...
.
... ... ..
... .
. ..
. ...
.
... .
... ... . ...
(A) 24 (B) 28 (C) 32 ...
...
...
... ...
... ..
.. . ..
...
...
. .
... ... ... ...
... ... ...
(D) 36 (E) 40 ...
....
..... ....
.. ....
. ...
.....
...
....
.....
.....
......
....... ....
15◦
..
.. .
....
......
......
.....
..........
........... . ....
...........
. .
.........................................
.
....

8. What is the remainder when 22010 is divided by 7?

(A) 1 (B) 2 (C) 3 (D) 4 (E) 5

9. The areas, in square centimetres, of three rectangles are given.


...............................................................................................................................................................................................................
... .. ..
.. ... ...
... ... ...
.... ... ...
... ... ...
... .
. ...
...
...
...
70 ..
.
..
.
.
.
25 ...
...
... .
. ...
... .
. ...
.
....................................................................................................................................................................................................................................
.
.
.............................................................................................................................................................................................................. ...
...................................................................................................................................................................................................... ...
............................................................................................................................................................................................................ ...
........................................................................................................................................................................................................ ...
.............................................................................................................................................................................................................
..............................................................................................................................................................................................................
.....................................................................................................................................................................................................
. .
20 ...
...
...
. . . . . . . . . .
................................................................................................................................................................................................
..................................................................................................................................................................... ..
............................................................................................................................................................................................................................................................

What is the area, in square centimetres, of the shaded rectangle?

(A) 36 (B) 48 (C) 56 (D) 60 (E) 70

1 1 1
10. If = + , the value of x is
6 2 x
1
(A) −3 (B) (C) 3 (D) 4 (E) −4
3
I3

Questions 11 to 20, 4 marks each

P
......
... ...
11. The perimeter of the equilateral triangle .
....
... .....
...
...
... ...
...
P QR is 48 cm. ..
.
....
.
...
...
...
.
.. ...
.. ...
What is the perimeter, in centimetres, of U ...
.......
.. ....
. ...
...
...
...
.
. ... ...
.. ...
the parallelogram P ST U ? .
.
...
.
.
..
. ....
....
....
...
...
...
... . ...
..
. ... ...
... .... ...
.
(A) 16 (B) 20 (C) 24 ..
.
..
.... ....
....
....
...
..
......
... .....
S
...
. . ... ...
... ... ..
. . .. ...
(D) 32 (E) 36 ...
...
...
. ...
...
...
... ....
...
.. ...
...
...
.. ... .. ...
...
. .
... . ...
. ...
...
. ...... ...
......................................................................................................................................................................................

R T Q

12. I am going to the shop with $4.20 to spend on my favourite chocolates, hazelnut
truffles at 30c each and orange truffles at 50c each. I do not want to buy more
than twice as many of one as the other. Apart from that, I want to buy as many
truffles as I can. How many is that?

(A) 10 (B) 11 (C) 12 (D) 13 (E) 14

13. Which of the following numbers cannot be expressed as the sum of two or more
consecutive positive integers?

(A) 12 (B) 13 (C) 14 (D) 15 (E) 16

14. Three rectangles are lined up horizontally as shown. The lengths of the rectan-
gles are 2 cm, 4 cm and 8 cm respectively. The heights are 1 cm, 2 cm and 4 cm
respectively. A straight line is drawn from the top right-hand corner of the largest
rectangle to the bottom left-hand corner of the smallest rectangle.
.....................................................................................................................................
......... .......... .............................. .
..........................................................................................................................................................................................
......... .......... ....................... .
....................................................................................................................................................................
..............................................................................................
......... .......... ...........
.......................................................................................................................................
.......................................................................
......... .......... . .
.....................................................................................................
...................................................
. . . . . . . . . . . . . . . ..
.................................................................
.............................
............................................................ .................................
. . . . . . . . . . . . . . . . . . . . . . . . . . . . . . . ..
..........................................................................................................
............................................................
...................... .
............................................................................
............... .
..... .......................................
....................................................... .................................................
.............................. ...............
..... .
................................................................
........................
..............
............

What is the area, in square centimetres, of the shaded region?

(A) 10 (B) 12 (C) 14 (D) 18 (E) 21

15. The value of (123456785) × (123456782) − (123456783) × (123456784) is

(A) −2 (B) −1 (C) 0 (D) 1 (E) none of these


I4

16. A three-digit number has all digits odd. How many such numbers are divisible by
three?

(A) 29 (B) 36 (C) 39 (D) 40 (E) 41

.......................................
17. Two squares are drawn inside a circle with centre .........
............
.........................................................................................
.........
.
........ .... . ....
.......................................................................................................................
.
O. The larger square touches the circle at each of .
.......
.... ...
.
..
. .
... .... ...........
.
..
. . .
. ... ... ....
... .... .... ... ... ....
its corners. The smaller square touches the circle ...
.... ...
...
...
...
..
....
.
.
....
...
...
...
... ...
...
... ... ... ...
at two of its corners and one of its sides passes .... ... .
. .
. ...

.
... ... .
. ..
... . . ...
... ..... ..... .... .... ...
... .. .. .
. .
. ...
through the centre of the circle as shown. ...
...
...
...
.
.
..
.................................................................... ..
.
..
.
.. ..
..
..
.
What is the ratio of the area of the larger square ...
...
...
.
..
.
.
....
O .
.
.
.
....
.
...
.
.
.

... ... ... ...


...
... .... ... ...
to that of the smaller square? ... ..
... ...
..... ..
. ....
.
.... ....
. ..
√ ........................................................................................................................
.....
...... .
.....
...
(A) 3 : 1 (B) 5 : 2 (C) 5 : 2 ......
.......
.........
.........
.......
.
.....
..

√ ...............
................................

(D) 2 : 1 (E) 3 : 6

18. How many integers n from 2 to 10 inclusive have the property that the sum of any
consecutive n positive numbers is odd?

(A) 0 (B) 1 (C) 2 (D) 3 (E) 4

19. The side length of the grid squares in the figure is 1 cm.
.................................................................................................................................................................
... ... ... ... ... ... .
... ... ... ... ...... ...
... ...... ...
... ... ... ... ... .......... ..
... ... ... ... .......... ...
... . .. . .. .
..............................................................................................................................................................................
.. .. .. ........
... . . . .....
.
... . . . . ....
.... .... .... .... .................................... ...
... ... ... . .. . . . . . . .. ...
... . . ..............................................
. . ....
........................................ . ...
... ...
. ..
. ..... . .
....................................................................................................................................................................................................
... ..
. .. ..........................................................................
. ....
.... .... .................................................................
. ...
... .... . . . . . . . . .... . . . . . . . ... ....
...
.. .
. .
..
............................................................................................... ..
... . . .
............................................................................. .
.
. ..
.. ..
. .. . . . . . . . . . . . . ... . . . . . . . ..
............................................................................................................................................................................................................
... . . ..
. .
. .
. .
. ...
... ..
........ .
.
.
.
.
. .
.
. ...
.
.... .
... ...... .... .... .... .... ...
... .......... .. ... ... ... ...
........... .... .... .... .... ...
........................................................................................................................................................

What is the area, in square centimetres, of the shaded triangle?


120 111 116 125 121
(A) (B) (C) (D) (E)
40 40 40 40 40

20. The 5-digit number a986b, where a is the first digit and b is the units digit, is
divisible by 72. What is the value of a + b?

(A) 9 (B) 10 (C) 12 (D) 13 (E) 15


I5

Questions 21 to 25, 5 marks each

21. Snugglepot and Cuddlepie were addicted to the video game Jabberwocky. One
morning each of them won 70% of their games. That afternoon, they played the
same number of games as each other and each won them all. Snugglepot’s winning
percentage for the day rose to 85% and Cuddlepie’s to 90%. The minimum possible
total number of games of Jabberwocky they could have played that morning was

(A) 10 (B) 20 (C) 30 (D) 60 (E) 70

22. There is a point X inside a square P QRS such that P X = 1, QX = 2 and triangles
P XQ and P XS are congruent. What is the area, in square units, of the square?

1+ 7 √ √
(A) (B) 4 (C) 4 − 7 (D) 4 + 7 (E) 5
2

23. In a race, each of four greyhounds runs at its own constant speed. They all start
at the same point on a circular track and at 30 seconds into the race, while all are
still on their first lap, they have spread out so they are at four corners of a square.
How many seconds into the race will it be when they are next at the four corners
of a square?

(A) 60 (B) 90 (C) 150 (D) 210 (E) 240

24. A ball starts rolling from the centre of a 3 m × 4 m room in a direction of 45◦ to
the walls. It rolls in a straight line except when it bounces off a wall and it does
that at an angle of 45◦ . .......................................................................................................................................
... ... ... ....
... ........ ........
... ...... ..... ...
.......... ..... ...
.......... ..... ...
.....
... ...... ..... ...
... ...... ..... ...
... ...... ..... ...
..... .....
... ..... ...... ...
...
...
...
.....
......
..
• .. ...
...
...
... .....
...... ...
..... ..... ...
.. ..... ...
.....
...
.... ✒
°
.....
.....
.....
...
...
..
°
...
......................................................................................................................................

After it has travelled 20 m, how many times has it bounced off a wall?

(A) 5 (B) 6 (C) 7 (D) 8 (E) 9

25. There are sixteen different ways of writing four-digit strings using 1s and 0s. Three
of these strings are 1010, 0100 and 1001. These three can be found as substrings
of 101001. There is a string of nineteen 1s and 0s which contains all sixteen strings
of length 4 exactly once. If this string starts with 1111, the last four digits are

(A) 1110 (B) 0000 (C) 0110 (D) 1010 (E) 0111
I6

For questions 26 to 30, shade the answer as an integer from 0 to 999 in


the space provided on the answer sheet.

Question 26 is 6 marks, question 27 is 7 marks, question 28 is 8 marks,


question 29 is 9 marks and question 30 is 10 marks.

26. How many whole numbers less than 2010 have exactly three factors?

27. Two 10 × 18 × ` blocks are placed on either side of a cylinder of length ` to stop it
from rolling. One block has a 10 × ` face on the ground while the other block has
an 18 × ` face on the ground. The block on the left sticks out 4 units more than
the one on the right. ....................................... ........ .........
....... .......
...... ......
..
......
.
.....
.....
..
.... .....
...
.... ...
...
. ...
.. ...
. ...
.... ...
... ...
... ...
... ...
... ..
... ...
... ..
.
... .
.
... ...........................
... ....... ..
... ..... ...
... .. .. ...
... .
.... ..... ...
.................................................. . ..
.
... . .... ...... .
... ...
... .... .......... .....
... ... ...
... ... ......
....... ......... ... ...
... ... ......... ..... . ..
.....................................................................................................................................................................................................................

What is the radius of the cylinder?

28. A number n is the sum of the squares of its four smallest divisors. What is the
smallest possible value of n?

29. I have a list of thirty numbers where the first number is 1, the last number is 30
and each of the other numbers is one more than the average of its two neighbours.
What is the largest number in the list?

30. There are many towns on the island of Tetra, all connected by roads. Each town
has three roads leading to three other different towns: one red road, one yellow
road and one blue road, where no two roads meet other than at towns. If you
start from any town and travel along red and yellow roads alternately (RYRY...)
you will get back to your starting town after having travelled over six different
roads. In fact RYRYRY will always get you back to where you started. In the
same way, going along yellow and blue roads alternately will always get you back
to the starting point after travelling along six different roads (YBYBYB). On the
other hand, going along red and blue roads alternately will always get you back to
the starting point after travelling along four different roads (RBRB). How many
towns are there on Tetra?
Intermediate 2010 Answers
Question Answer
1 D
2 D
3 D
4 A
5 C
6 C
7 A
8 A
9 C
10 A
11 D
12 B
13 E
14 C
15 A
16 E
17 B
18 D
19 E
20 D
21 C
22 D
23 B
24 E
25 E
26 14
27 25
28 246
29 226
30 24
A u s t r a l i a n M a t h e ma t i c s C o m p e t i t i o n
an activity of the australian mathematics trust

t h u r s d ay 4 A u g u s t 2 011

intermediate Division Competition Paper


australian School Years 9 and 10
time allowed: 75 minutes

Instr uct ions a nd I nf or m ati on


GENERAL
1. Do not open the booklet until told to do so by your teacher.
2. NO calculators, slide rules, log tables, maths stencils, mobile phones or other calculating aids are
permitted. Scribbling paper, graph paper, ruler and compasses are permitted, but are not essential.
3. Diagrams are NOT drawn to scale. They are intended only as aids.
4. There are 25 multiple-choice questions, each with 5 possible answers given and 5 questions that
require a whole number answer between 0 and 999. The questions generally get harder as you work
through the paper. There is no penalty for an incorrect response.
5. This is a competition not a test; do not expect to answer all questions. You are only competing
against your own year in your own State or Region so different years doing the same paper
are not compared.
6. Read the instructions on the answer sheet carefully. Ensure your name, school name and school year
are entered. It is your responsibility to correctly code your answer sheet.
7. When your teacher gives the signal, begin working on the problems.

THE ANSWER SHEET


1. Use only lead pencil.
2. Record your answers on the reverse of the answer sheet (not on the question paper) by FULLY
colouring the circle matching your answer.
3. Your answer sheet will be scanned. The optical scanner will attempt to read all markings even
if they are in the wrong places, so please be careful not to doodle or write anything extra on
the answer sheet. If you want to change an answer or remove any marks, use a plastic eraser
and be sure to remove all marks and smudges.

INTEGRITY OF THE COMPETITION


The AMT reserves the right to re-examine students before deciding whether to grant official status
to their score.

©AMT Publishing 2011 amtt limited acn 083 950 341


Intermediate Division

Questions 1 to 10, 3 marks each

1. The value of 2011 − 1102 is

(A) 1111 (B) 1191 (C) 1001 (D) 989 (E) 909

2. In the diagram, the value of x is ..


.........
..... ......
........
........ ..... ..............
.
.. ... ...
...... ...
..... ...
.....
...... ...
...
....... ...
...
...
..... ...
..
....... ...
...
..
...... ...
...
..... ...
.
........ ...
...
..
...... ...
......... ...
...
....
127◦ ..
.
.....
.
... .
...... ...
...
..
x◦
..........................................................................................................................................................................................................................................................................................................................
.
.

(A) 143 (B) 127 (C) 90 (D) 153 (E) 37

3. The value of 14 ÷ 0.4 is

(A) 3.5 (B) 35 (C) 5.6 (D) 350 (E) 0.14

4. Which of the following could be the graph of y = 2x + 1?

(A) y
✻ (B) y
✻ (C) y

..
...
...
... ....
..
.
. ......
...
....... .......
...
....... .......
...
.......
....... ..
........
.
... ....... .
......
. ......
✲x ✲x ✲x
... .......
...
....... .......
...
....... .......
...
.......
....... ...
........
.
... ....... ......
..
.
. ....... ......
... ....... .......
... .......
.. .......
...
.....
...
...
...

(D) y
✻ (E) y

......
......
......
......
......
...... ......
...... ......
...... ......
......
✲x ✲x
...... ......
...... ......
...... ......
...... ......
...... ......
...... ......
. ......
......
......
......
......
......
......
...

5. The expression 8x − 4y − 3x + 2y equals

(A) 4x − y (B) 5x − 2y (C) 5x − 6y (D) 11x − 2y (E) 11x − 6y


I2

1
6. By what number must be divided to obtain 4 as a result?
3
1 1 1
(A) (B) 6 (C) 1 (D) (E) 12
12 3 4

7. Which one of the following is not equal to 39 ?

(A) (33 )3 (B) 33 × 33 × 33 (C) 273 (D) 93 × 27 (E) 94

8. The numbers represented by points R and P on the number line below are multi-
plied. Which point would best represent the product of these two numbers?

M
. ...
. ..
S R P ... T
. ..
..............................................................................................................................................................................................................................................................................................................................................................................................
... ... ...
N
.
0 1 2

(A) M (B) N (C) P (D) S (E) T

9. P QRS is a trapezium in which


2
P Q = 2 units and RS = 3 units. P ...............................................................................................................................................................................................................................................................Q
... ....... . . . . . . . . ........... . . . ......
.. ...................... . . . . ......... ....
What fraction of the trapezium is ...
....
.............................................
..........................................
....................... . ...............
...... . . . . . . ..... . . . . . ...
... .......................................
....... ............................
shaded? ...
...
...
...................................
....... . . . . . . . . . . . ...
...............................
...............................
... ............................
1 1 1 ...
...
....... . . ..............
.......... . . . . . ....
.......................
(A) (B) (C) ...
...
....... ...............
...... ..............
....... . . . . ....
5 4 3 ...
...
..................
.................
....... .......
....... . ...
... .............
2 1 ... ...........
........................................................................................................................................................................................
.. .
(D) (E) S R
5 2 3

10. An 8 × 8 × 8 hollow cube is constructed from 1 × 1 × 1 cubes so that its six walls
are 1 cube thick. The number of 1 × 1 × 1 cubes needed to make the hollow cube
is

(A) 169 (B) 296 (C) 298 (D) 384 (E) 512

Questions 11 to 20, 4 marks each

11. In my neighbourhood, 90% of the properties are houses and 10% are shops. 10%
of the houses are for sale and 30% of the shops are for sale. What percentage of
the properties for sale are houses?
1
(A) 9% (B) 80% (C) 33 % (D) 75% (E) 25%
3
I3

12. P QRS is a square. T U V W is a smaller square P ..............................................................................................................................................................................................................................Q


...........................................................................
............................................................................
placed inside as shown with P R = 2T V . ............................................................................................................
... . . ............ ......... . . . . . . ................. . ...........
... . .......
................................................................................................................
.. .
............................................................................................................................................
The ratio of the shaded area to the area of the ........................ .......
T
............................ .......
U
......................
..............................
.............................. .....
..... ..............................
........................
square P QRS is ........................
............................
..............................
.....
.....
.....
.....
............................
..............................
......................... .....
..... ........................
........................... ..... ............................
.............................. ..............................
(A) 2 : 3 (B) 3 : 4 (C) 1 : 3 .........................
...........................
.....
..... .
..... ......................................
..... .. ......... . ......
.
.......................... ........ . ......... ...
(D) 1 : 2 (E) 2 : 5 W ...............................................................................................................
V
...........................................................................................................................................
..............................................................................................................
....................................................................................................
.......................................................................................
..............................................................................................................
...............................................................................................................................
S R

13. The numbers on the six faces of this cube are consecutive even numbers.

If the sums of the numbers on each of the three pairs of opposite faces are equal,
find the sum of all six numbers on this cube.
(A) 196 (B) 188 (C) 210 (D) 186 (E) 198

14. The positive integers are arranged in a zigzag fashion across five rows as follows:
A 1 9 17
B 2 8 10 16 18
C 3 7 11 15 19
D 4 6 12 14 .
E 5 13 .
In which row will 2011 appear?
(A) A (B) B (C) C (D) D (E) E

15. Two tourists are walking 12 km apart along a flat track at a constant speed of
4 km/h. When each tourist reaches the slope of a mountain, she begins to climb
with a constant speed of 3 km/h.




?k

m✡
✑m ✡

✰k ?
✛ 12 km ✲
12 12
kmkm

What is the distance, in kilometres, between the two tourists during the climb?
(A) 16 (B) 12 (C) 10 (D) 9 (E) 8
I4

16. The six faces of a dice are numbered −3, −2, −1, 0, 1, 2. If the dice is rolled twice
and the two numbers are multiplied together, what is the probability that the
result is negative?
1 1 11 13 1
(A) (B) (C) (D) (E)
2 4 36 36 3

17. A 36 cm by 24 cm rectangle is drawn on 1 cm grid paper such that the 36 cm side


contains 37 grid points and the 24 cm side contains 25 grid points. A diagonal of
the rectangle is drawn. How many grid points lie on that diagonal?

(A) 10 (B) 12 (C) 13 (D) 15 (E) 21

18. Three people play a game with a total of 24 counters where the result is always
that one person loses and two people win. The loser must then double the number
of counters that each of the other players has at that time.
At the end of three games, each player has lost one game and each person has 8
counters. At the beginning, Holly had more counters than either of the others.
How many did she have at the start?

(A) 9 (B) 11 (C) 13 (D) 16 (E) 24

19. Mary has 62 square blue tiles and a number of square red tiles. All tiles are
the same size. She makes a rectangle with red tiles inside and blue tiles on the
perimeter. What is the largest number of red tiles she could have used?

(A) 62 (B) 182 (C) 210 (D) 224 (E) 240

20. An isosceles triangle has a horizontal base of length 12 centimetres. It is divided


into four equal areas by three parallel lines as shown.
...........
...... .. ......
...... .... ...........
...... ......
..
..
.. .... ......
...... ... ......
...... ... .....
. ..
.......
. . ......
....
...... .
. ...
..
... ...
.
.
.
. .... ..........
..
.. .
. ... ........
..
....
. .
.
. .
. .
. .....
.. .
...... .... .... ...
......
...... ... ... ....
......
..
........ .
. . .
.
......
......
. .
... . .
. . ......
..
.. .
. .
. .
. ......
.
.....
. .
.
. .
. .
.
. ......
.
.... . .
. . ......
.... .
. .
. .
.
...
.... .
.
. .
. .
.
.
......
. .
... . .
. . ......
...
. .
. .
. .
. ......
.
...... .
.
. .
.
. .
.
. ......
.
.... . . . .....
.. . . .
....................................................................................................................................................................................................................................................................
.
. . . .
✛ x cm ✲

What is the value of x?


√ √
(A) 3 2 (B) 4 (C) 4.5 (D) 3 (E) 3 3
I5

Questions 21 to 25, 5 marks each

21. Of the staff in an office, 15 rode a pushbike to work on Monday, 12 rode on Tuesday
and 9 rode on Wednesday.
If 22 staff rode a pushbike to work at least once during these three days, what is
the maximum number of staff who could have ridden a pushbike to work on all
three days?

(A) 4 (B) 5 (C) 6 (D) 7 (E) 8

22. I drive a distance of 200 km around the city and my car’s average speed is 25 km/h.
How far do I then need to drive at an average speed of 100 km/h to raise my car’s
average speed for the whole time to 40 km/h?

(A) 400 km (B) 200 km (C) 150 km (D) 120 km (E) 100 km

23. How many 3-digit numbers can be written as the sum of three (not necessarily
different) 2-digit numbers?

(A) 194 (B) 198 (C) 204 (D) 287 (E) 296

24. A circle of radius 90 units and a circle of radius 40 units are tangent to each other
and tangent to two lines as shown in the diagram below.
.........................................
......... ........
...... .............
...... .............
..
...... ..... ........
...
. ... ..........
... ... ........
. ... ........
..
. ... ........
.... ... .............................
... ..............
...
...
...
...
. ...
...
... ......
..
.....
.....
.
........
............
... ........
... .......
...
...
........
........
...
...
...
...
... 90
...
...
.
.
.
..... .
.
..
..
....
.. ....
..
.
...
. ...
..
. ..
...
....
.
........
........
........
........
..........
........
.. ... ... .
....
.....
......
.......
.
.....
.. .
...... .
.........
40 ...
......
.
.
.
...
.
.. ... ..... ........
........
........
........
......... ......... ........... . .
...... .
.......... ......... ....
. ..
.............................................................................................................................................................................................................................................................................................................................
. . . . .

X Y
What is the distance XY ?

(A) 120 (B) 180 (C) 216 (D) 234 (E) 260
I6

25. An arrangement of numbers has different differences when the differences between
neighbours are all different. For example, the numbers
1 4 2 3
have differences 3, 2 and 1 − all different.
If the numbers from 1 to 6 are arranged with different differences, and with 3 in
the third position,
3
what is the sum of the last three digits?

(A) 12 (B) 13 (C) 14 (D) 15 (E) 16

For questions 26 to 30, shade the answer as an integer from 0 to 999 in


the space provided on the answer sheet.

Question 26 is 6 marks, question 27 is 7 marks, question 28 is 8 marks,


question 29 is 9 marks and question 30 is 10 marks.

26. The first digit of a six-digit number is 1. This digit 1 is now moved from the first
digit position to the end, so it becomes the last digit. The new six-digit number is
now 3 times larger than the original number. What are the last three digits of the
original number?

27. The diagram shows the net of a cube. On each face there is an integer: 1, w, 2011,
x, y and z. ......................................... ...
.... ...
... ...
..
...
...
w ...
...
...
... ...
..............................................................................................................................................................
... ... ... ... ...
... ... ... .... ....

x ...
...
...
...
...y
... 2011
....
..
...
...
z
...
...
...
...
...
...
...
... ... ...
....................................................................................................................................................................
... ..
... .
.
... ...
... ....
...
....
1 ...
...
.... ..
........................................

If each of the numbers w, x, y and z equals the average of the numbers written on
the four faces of the cube adjacent to it, find the value of x.
I7

28. Two
√ beetles sit at the vertices A and H of a cube ABCDEF GH with edge length
40 110 units. The beetles start moving simultaneously along AC and HF with
the speed of the first beetle twice that of the other one.

✈s
A ...............................................................................................................................................
.. .. . ..
D
.... ....... ...... ..... ... .. .........
... ......... ... .....
. ...... .. .....
.....
... .....
..... ...... .. .. .....
... ..... .
... ...... .. .. .....
.....
... .....
..... .... ...... .
. .....
... ..... ..
.... ...... .....
... ..... .. . ...... . ...
... ..... . ..... ... ........
..... . ... ..... ....
... ....................................................................................................................................
.
...
... B ...
..
..
..
..
...
C
... .. .. ...
... ..... .. ...
... ... .. ...
... ... .. ...
... ... .. ...
... ... .. ...
... ... .. ...
... ... .. ...
... ... .. ...
... ... .. ...

s✈
... ... .. ....
... ... .. ..
...
E .......... ... ... ... ... ... ... ... ... ..... ... ... ... ... ... ... ... ... ... ... ... ... ... ........
.....
.....
.....
...
.
.
... ..
..
.. ... ...
...
...
H ...
...
..... ... ..
.... ..... ...
..... . ..
..
.. . ...
..... ... . ...
..... ... ..... ..... ...
..... ... ..
. . ...
..... ... ...
..... . .
..... .... ...
..... .... . .
. ... ....
.
..... ... . .... ..
....... .....
.......................................................................................................................................
F G
What will be the shortest distance between the beetles?

29. In the diagram, 4P QR has an area of 960 square units. The points S, T and U
are the midpoints of the sides QR, RP and P Q, respectively, and the lines P S,
QT and RU intersect at W .

....
R
..........
..... .. ..
..... .. ...
.
... ..... .... .....
.. ...
.....
..... ... ...
..... ... ...
...
..... ..
..
. . ....... ..
...
...
...
..... .
.. ...
...
..
..... ..
. ...
.... . ...
....
T
.
..
.....
.
.
...
.... ................ ...
.
.
.
.
.
.......
.
N ..
...
.....
..
..............
S
..
..... ..........
.. .. ... .. ..... ..
....
........
. ...
...
.... . ............ . .
. ................ . . ...
..
.. ......
. .. . ...
..
..... ... ...................... . ...
..... ................................................. ..... ...
.
....
.
..
.. . .L ..
............. ... ... ... .......... ... ...
... M


..
.. .......... .
..
... ....... ... ... ... ... ... .................. .
.. ...
.... ......

.... ....
...... . . .
...
. ...
.
...... .
...
............. ..
.
..........
..... ...
..... . ........ . .......... ..
. .. ....... .
. ..
............ .....
.
.
. .. . .... ................. W..
. .. .
.
...................................................................................................................................................................................................................................................................
P .
...
Q
U
The points L, M and N lie on P S, QT and RU , respectively, such that
P L : LS = 1 : 1, QM : M T = 1 : 2 and RN : N U = 5 : 4.
What is the area, in square units, of 4LMN ?

30. A 40 × 40 white square is divided into 1 × 1 squares by lines parallel to its sides.
Some of these 1 × 1 squares are coloured red so that each of the 1 × 1 squares,
regardless of whether it is coloured red or not, shares a side with at most one red
square (not counting itself). What is the largest possible number of red squares?
Intermediate 2011 Answers
Question Answer
1 E
2 A
3 B
4 C
5 B
6 A
7 E
8 D
9 D
10 B
11 D
12 B
13 E
14 C
15 D
16 E
17 C
18 C
19 C
20 A
21 D
22 B
23 B
24 C
25 A
26 857
27 805
28 440
29 80
30 420
A u s t r a l i a n M a t h e ma t i c s C o m p e t i t i o n
an activity of the australian mathematics trust

T H U R S D AY 2 A U G U S T 2 01 2
NAME

INTERMEDIATE DIVISION COMPETITION PAPER


AUSTRALIAN SCHOOL YEARS 9 AND 10
TIME ALLOWED: 75 MINUTES

INSTR UC T IONS A ND I NF OR M ATI ON


GENERAL
1. Do not open the booklet until told to do so by your teacher.
2. NO calculators, slide rules, log tables, maths stencils, mobile phones or other calculating aids are
permitted. Scribbling paper, graph paper, ruler and compasses are permitted, but are not essential.
3. Diagrams are NOT drawn to scale. They are intended only as aids.
4. There are 25 multiple-choice questions, each with 5 possible answers given and 5 questions that
require a whole number answer between 0 and 999. The questions generally get harder as you work
through the paper. There is no penalty for an incorrect response.
5. This is a competition not a test; do not expect to answer all questions. You are only competing
against your own year in your own State or Region so different years doing the same paper
are not compared.
6. Read the instructions on the answer sheet carefully. Ensure your name, school name and school year
are entered. It is your responsibility to correctly code your answer sheet.
7. When your teacher gives the signal, begin working on the problems.

THE ANSWER SHEET


1. Use only lead pencil.
2. Record your answers on the reverse of the answer sheet (not on the question paper) by FULLY
colouring the circle matching your answer.
3. Your answer sheet will be scanned. The optical scanner will attempt to read all markings even
if they are in the wrong places, so please be careful not to doodle or write anything extra on
the answer sheet. If you want to change an answer or remove any marks, use a plastic eraser
and be sure to remove all marks and smudges.

INTEGRITY OF THE COMPETITION


The AMT reserves the right to re-examine students before deciding whether to grant official status
to their score.

©AMT Publishing 2012 amtt limited acn 083 950 341


Intermediate Division

Questions 1 to 10, 3 marks each

1. The value of 8 × 3.3 is

(A) 24.24 (B) 24.4 (C) 25.4 (D) 26.24 (E) 26.4

2. Sally has $20 of her pocket money left after 3 weeks, having spent just $1 on a
drink. How much pocket money does she get each week?

(A) $5 (B) $7 (C) $9 (D) $20 (E) $21

Q ....
.... ......
..... ..........
3. In the diagram, the size of 6 P QR is ..
.
.....
....
.....
.....
.....
..... .....
......
. .....
... .....
(A) 40◦ (B) 50◦ (C) 60◦ ..
.. .
..
.
.
.
...
.
. .....
.....
.....
.....
.....
. .....
...
. .....
(D) 70◦ (E) 80◦ .
...
.
.
.
...
.
.
.
...
. .....
.....
.....
.....
..
. .....
..
.... .....
.....
..
.... .....
..
.... .....
.....
...
120◦ ..
.. .
.
.....
.
.
.. .....
130◦
.....
...
.............................................................................................................................................................................................................................................................

P R

4. Three-fifths of a number is 48. What is the number?

(A) 54 (B) 60 (C) 64 (D) 80 (E) 84

1
5. By what number must 6 be divided to obtain as a result?
3
1 1
(A) 18 (B) (C) (D) 2 (E) 9
2 18

1 1 1
6. The average of the five numbers x, 1, , and is 1. The value of x is
2 3 4
1 2 11 25 35
(A) (B) (C) (D) (E)
5 3 5 12 12
I2

........................................................................................................................................
7. In the diagram, P QRS is a square. P ... ......
... ...... ..
....... ...
Q
..... ......... ....... ..
..... ....... ...
..
... ..... .
............ ...
..... ....
The value of x is ....
...
...
.....
.....
.....
..... ..
.... ..... ..
... .
....... .
. ...
...
...
.. ..... ........... ..
...
...
... ........ .
.. .
..........
. .
..... ........
.....
x◦ ....
...
... ......... ...... ...
.. .
.. ..
..... ...
... ..........
(A) 45 (B) 60 (C) 67.5 ..
...
........ .
........... .......
..
.....
.
..
...
...
...
..
....... ....
. .....
....... ...
.
....... ... ...
(D) 75 (E) 82.5 x◦ ...
...
...
..
.....
.......
.......
...
...
... ..
..... ...
... .....
..... ...
...
... ..... ....
..... ..
.
. .......
..............................................................................................................................
S R

8. Five positive integers have a mean of 10, a median of 10 and only one mode, which
is 12. What is the difference between the largest and smallest of these numbers?
(A) 3 (B) 5 (C) 6 (D) 7 (E) 8

9. If 750 × 45 = p, then 750 × 44 equals


(A) p − 45 (B) p − 750 (C) p − 1 (D) 44p (E) 750p

10. I can ride my bike 3 times as fast as Ted can jog. Ted starts 40 minutes before me
and then I chase him. How long does it take me to catch Ted?
(A) 20 min (B) 30 min (C) 40 min (D) 50 min (E) 60 min

Questions 11 to 20, 4 marks each

11. If p% of q is k, then q% of p is
k pq pk qk
(A) (B) (C) (D) (E) k
100 200 100 100

12. On one side of each of the five coins below there is a number and on the other side
there is a shape.
.............................. .............................. .............................. .............................. ..............................
....... ...... ....... ...... ....... ...... ....... ...... ....... ......
..... .... ..... .... ..... .... ..... .... ..... ....
.... ... .... ... .... ... .... ... .... ...
.... ... .... ... .... ... .... ... .... ...
... ... ... ... ... ... ... ......
. ... ... ...
. .. ...
.... ... .... ... .... ... .... ... .... ... .... .
........ ......... ...
...
...
...
...
2 .....
...
..
...
...
...
...
3 .....
...
..
...
...
...
...
4 .....
...
..
...
...
...
... ......
.. ...
. ...
...
........................... .....
...
..
...
...
...
...
..
.
..........................
...

.....
...
..
... ..
. ... ..
. ... ..
. ... ..
. ... ..
.
..... ... ..... ... ..... ... ..... ... ..... ...
...... .... ...... .... ...... .... ...... .... ...... ....
.......... ...... .......... ...... ......... ...... ......... ...... ......... ......
...................... ...................... ....................... ....................... .......................

P Q R S T
Peter is told that if there is a triangle on one side of a coin then there is an even
number on the other. Which of the following is the fewest coins that Peter can
turn over from the five to check this?
(A) S (B) P and R (C) Q and S (D) P , Q, R and S (E) all coins
I3

13. The architecture of Federation Square in Melbourne is based on frames as shown


in which a large triangle is subdivided into 5 identical triangles, each similar to the
large triangle. .
........ .... ..
....... ..........
.......
......... ..... .....
..
...
.. ........ ... ...
.
....... ... ....
.
....
........ .
. ...
.... ... ...
.......
....... ... ...
.....
........ ...
.
...
...
... ..
.
................................................................................................... ...
.
......... ... .
......... .. ...
....
....... ..
. ...
....... ..
. ...
...
..
....... .... .
...
..... ... ...
....... .
. ..... . .
. ...
.
... . .
.... .
....
....... ...
. .......... ... ...
...
....... .
. .
....
..... .... ...
...
......
. ... ...... ...
.... . .... ...
....
...... ..
. ..
........ ..
. ...
........... ... ........
..
... ...
. .
.............................................................................................................................................................................................................................................................

If the shortest side of one of the smallest triangles is 1 m, how many metres of
framing are required to construct the whole shape?
√ √ √ √
(A) 20 (B) 8 + 4 5 (C) 10 + 4 5 (D) 12 + 4 5 (E) 15 + 5 5

14. If a : b = 3 : 2 and a + 3b = 27, what is the value of a + b?

(A) 5 (B) 9 (C) 13 (D) 15 (E) 21

15. This sheriff’s badge has ten equal sides, five 60◦ angles and five equal reflex angles.
..
.......
... ...
... ............................
.....
.
... ◦......
60
...
... ...
... ◦ .....
x
...
..
.. ... .
...... . . . . . . ...
...
....... ................... ...........................
.
..................... .. ....... ...................
...... . .
..... .... ................... ....... ....
..... .. . .... .. .....
..... ... .... . . .....
..... ......
.
.....
SHERIFF
.....
..... . .
... ... ..... .. .. .....
. ......
.
.....
. .... ..................... ..... ....
.... .....
........ ....
.. .
...
.. ................... ...
... ...
... .....
....... .............
...
.... .... .... .. ....... ..
... ............. ....... ....
....... ..
.......... ......

The value of x is

(A) 108 (B) 132 (C) 135 (D) 138 (E) 140

16. The shape shown is formed from four identical arcs, each a quarter of the circum-
ference of a circle of radius 5 cm.
.....
................ .... ... ... ... ....
........ ....
...... ... ...
.
......
. ...
...
...
. ..
.... ... ..
.... ... ..
... ...
.....
.
..
.... ..... ..
... ...... .
.......
.... .......... ...
.................... ............
...... ..
.. .......
... ..
.
.
.. .....
.. ..... ..
.. ... ....
... .
.. ... ...
.. ... ...
... ... ....
...
.... ... .
...
......
..... .. .....
... ... ... ...............................

What is the area, in square centimetres, of the shape?


25π 25π
(A) 50 (B) (C) 25π − 25 (D) 100 − (E) 25π
2 2
I4

17. The number 2012 × 2013 × 2014 + 2013 is the cube of

(A) 2012 (B) 2013 (C) 2014 (D) 2112 (E) 2113

18. A partition of a positive integer is a way of writing the integer as a sum of at least
two positive integers. For example, the partitions of 4 are:

3 + 1, 2 + 2, 2 + 1 + 1 and 1 + 1 + 1 + 1.

How many partitions of 7 are there?

(A) 11 (B) 12 (C) 13 (D) 14 (E) 15

19. These five shapes all have the same area. Which one has the largest perimeter?
(A) a
(B) ....................................................b................................................................. (C)
.
... .....
....
...
...............................................................
...
...
...
..
.....
.....
.....
4c
... ... ... ..
.. .....................................................................................................................................
... ... ... ..... ...
.... ...
... ... ... ..... ... ...
... ... ... ..... ... ...
a ... ... a b ... .
..
...... c ... ... c
.
... ... ... ..... ... ...
..... .... ...
... ... ..... .........
... ... ... ..... ..................................................................................................................................
.....
........
.. ...
.................................................................
...
..
...
...
... .........
.
...
.....
.
.
4c
a ............
..

.................................
........
(D) .....
.....
..... ......
.....
....
... (E)
... ... .............................................................................................................................
...........
....
d
...
... .... 2e .....
.......
......
...

... ✲
................................................................................
.
..
...
... ...
...
.......
.......
... .. ... .......
... ..
e .......
... ..
.
. ... ......
... ... .......
...
..... ... ... .
...
........
..... ..... ... ......
...... ...... .......
..........
...................... ...... ... ............
.. ......
.......

20. Pippa made a litre of drink from apple juice and water in the ratio of 1 : 2. She
found the taste too strong so she made a litre again in the ratio 1 : 3, but found
this too weak. So she thought if she combined these two mixtures, it should be
about right. What is the ratio of apple juice to water in this new mixture?

(A) 2 : 5 (B) 2 : 7 (C) 5 : 12 (D) 7 : 17 (E) 7 : 24

Questions 21 to 25, 5 marks each

21. A courier company has motorbikes which can travel 300 km starting with a full
tank. Two couriers, Anna and Brian, set off from the depot together to deliver
a letter to Connor’s house. The only refuelling is when they stop for Anna to
transfer some fuel from her tank to Brian’s tank. She then returns to the depot
while Brian keeps going, delivers the letter and returns to the depot. What is the
greatest distance that Connor’s house could be from the depot?

(A) 180 km (B) 200 km (C) 225 km (D) 250 km (E) 300 km
I5

22. QRST is a trapezium in which QR k T S and QR : ST = 2 : 3.

Q ................................................................................................................................................................................ R
....... .. ...
.
...
....... .......
....... ....... ...
... ....... ....... ...
.. .......
.. ............... ...
. ....... .
..
. ....... .............. ...
...
... ... . . . ........ ...
..
. ........... .......
.......... ...
.. ....
...... .... ...
...
.
.
..
..........
. ..
..........
X ........
..
.........
....
...
...
... .....
...... .......
...
.
...
...
..... .......
... .
...
.. ...
........ .
... .............. ........ .....
.
.........................................................................................................................................................................................................
T .... S

If the area of 4XST is 18 square units, what is the area, in square units, of
4RT S?

(A) 24 (B) 28 (C) 30 (D) 34 (E) 36

23. If abc + ab + bc + ca + a + b + c = 104, and a, b and c are positive integers, then


a2 + b2 + c2 is equal to

(A) 49 (B) 51 (C) 54 (D) 56 (E) 60

24. A teacher has a class of twelve students. She thinks it would be a nice idea if they
change desks every day, so she has painted arrows on the floor from desk to desk.
Each desk has one arrow going to it and another going from it. Each morning,
the students pick up their books and move to the desk indicated by the arrow.
By choosing her arrows carefully, the teacher has arranged it so that the longest
possible time will pass before all the students are back in their original desks at
the same time. How many days is that?

(A) 30 (B) 35 (C) 42 (D) 60 (E) 72

25. The number 3333 can be expressed as the sum of 33 consecutive odd numbers. The
largest of these odd numbers is

(A) 3332 + 32 (B) 3331 + 32 (C) 3332 − 32 (D) 3331 − 32 (E) 3332

For questions 26 to 30, shade the answer as an integer from 0 to 999 in


the space provided on the answer sheet.

Question 26 is 6 marks, question 27 is 7 marks, question 28 is 8 marks,


question 29 is 9 marks and question 30 is 10 marks.
I6

26. Slim took a long road trip across Australia over a number of days (more than 1).
When he arrived at his destination, he noted that he had travelled exactly 2012
kilometres. On the first day he travelled a whole number of kilometres and each
subsequent day he travelled one more kilometre than the day before. What is the
largest distance, in kilometres, that he could have travelled on the first day?

27. Five consecutive positive integers, p, q, r, s and t, each less than 10 000, produce
a sum which is√a perfect square, while the sum q + r + s is a perfect cube. What
is the value of p + q + r + s + t?

28. A quadrilateral with sides 15, 15, 15 and 20 is drawn with each vertex on a circle.
Around this circle a square is drawn, with each side tangent to the circle. What is
the area, in square units, of this square?

29. In the grid shown, we need to fill in the squares with numbers so that the number
in every square, except for the corner ones, is the average of its neighbours. The
edge squares have three neighbours, the others four.
.................................................................................................................................................................................................
.... .. .. .. .. ..
... ... ... ... ... ...
... ... ... ... ... ...
+1000
...
...
...
...
...
...
...
...
...
... −1000 ...
...
... ... ... ... ... ...
... ... ... ... ... ...
..........................................................................................................................................................................................................
... ... ... ... ... ...
... ... ... ... ... ...
... ... ... ... ... ...
...
...
x ...
...
...
...
...
...
...
...
...
...
... ... ... ... ... ...
........................................................................................................................................................................................................
... ... ... ... ... ...
... ... ... ... ... ...
... ... ... ... ... ...
... ... ... ... ... ...
... ... ... ... .. ...
... ... ... ... .... ...
... ... ... ... ... ..
..........................................................................................................................................................................................................
... ... ... ... ... ...
... ... .. ... ... ...
... .... ... ... ... ...
... .. ... ... ... ...
... ... ... ... ... ...
... ... ... ... ... ..
.........................................................................................................................................................................................................
... ... ... ... ... ...
... ... ... ... ... ...
... ... ... ... ... ...
−1000 ...
...
...
...
...
...
...
... +1000 ...
...
...
...
... ... ... ... ... ...
... ... .... ... .... ..
...........................................................................................................................................................................................

What is the value of the number in the square marked x?

30. Terry has invented a new way to extend lists of numbers. To Terryfy a list such
as [1, 8] he creates two lists [2, 9] and [3, 10], where each term is one more than
the corresponding term in the previous list, and then joins the three lists together
to give [1, 8, 2, 9, 3, 10]. If he starts with a list containing one number [0] and
repeatedly Terryfies it he creates the list

[0, 1, 2, 1, 2, 3, 2, 3, 4, 1, 2, 3, 2, 3, 4, 3, 4, 5, 2, 3, 4, . . . ].

What is the 2012th number in this Terryfic list?


Intermediate 2012 Answers
Question Answer
1 E
2 B
3 D
4 D
5 A
6 E
7 C
8 B
9 B
10 A
11 E
12 C
13 C
14 D
15 B
16 A
17 B
18 D
19 E
20 D
21 B
22 C
23 D
24 D
25 A
26 248
27 75
28 540
29 200
30 9

©AMT Publishing 2012 amtt limited acn 083 950 341


Australian Mathematics Competition
sponsored by the Commonwealth Bank
an activity of the australian mathematics trust

A u s t r a l i a n M at h e mat i c s T r u s t

NAME

YEAR TEACHER

2013
INTERMEDIATE DIVISION
AUSTRALIAN SCHOOL YEARS 9 and 10
TIME ALLOWED: 75 MINUTES

INSTRUCTIONS AND INFORMATION


GENERAL
1. Do not open the booklet until told to do so by your teacher.
2. NO calculators, slide rules, log tables, maths stencils, mobile phones or other calculating aids are permitted.
Scribbling paper, graph paper, ruler and compasses are permitted, but are not essential.
3. Diagrams are NOT drawn to scale. They are intended only as aids.
4. There are 25 multiple-choice questions, each with 5 possible answers given and 5 questions that require a
whole number answer between 0 and 999. The questions generally get harder as you work through the paper.
There is no penalty for an incorrect response.
5. This is a competition not a test; do not expect to answer all questions. You are only competing against your
own year in your own country or Australian state so different years doing the same paper are not compared.
6. Read the instructions on the answer sheet carefully. Ensure your name, school name and school year are
entered. It is your responsibility to correctly code your answer sheet.
7. When your teacher gives the signal, begin working on the problems.

THE ANSWER SHEET


1. Use only lead pencil.
2. Record your answers on the reverse of the answer sheet (not on the question paper) by FULLY colouring the
circle matching your answer.
3. Your answer sheet will be scanned. The optical scanner will attempt to read all markings even if they are in
the wrong places, so please be careful not to doodle or write anything extra on the answer sheet. If you want
to change an answer or remove any marks, use a plastic eraser and be sure to remove all marks and smudges.

INTEGRITY OF THE COMPETITION


The AMT reserves the right to re-examine students before deciding whether to grant official status to their score.

©AMT Publishing 2013 amtt limited acn 083 950 341


Intermediate Division
Questions 1 to 10, 3 marks each

1. 2013 + 2014 + 2015 equals

(A) 642 (B) 2016 (C) 6022 (D) 6032 (E) 6042

2. In the diagram below, what is the value, in degrees, of angle x?

85◦

x◦

37◦

(A) 48 (B) 85 (C) 122 (D) 132 (E) 143

3. If every digit of a whole number is either a 3 or a 5, the number will always be

(A) divisible by 3 (B) divisible by 5 (C) prime (D) even (E) odd

4. The average of two numbers is twice the smaller number. The larger number is 12.
What is the smaller number?

(A) 2 (B) 3 (C) 4 (D) 6 (E) 8

5. The length of the base of a triangle is 3 times its perpendicular height and the area
of the triangle is 24 cm2 . The sum of its base length and its perpendicular height,
in centimetres, is

(A) 12 (B) 13 (C) 14 (D) 15 (E) 16

6. A regular icosahedron is a solid shape with twenty faces, where each face is directly
opposite another face. I label the faces from 1 to 20 so that, for all pairs of opposite
faces, the two labels in any pair always add up to the same number. What number
is on the face opposite the one labelled 8?

(A) 11 (B) 12 (C) 13 (D) 14 (E) 15


I2

7. If p = 4b + 26 and b is a positive integer, then p could not be divisible by

(A) 2 (B) 4 (C) 5 (D) 6 (E) 7

8. My two dogs were running on the beach when I called them back. The faster dog
was 100 m away and the slower dog was 70 m away. The faster dog runs twice as
fast as the slower dog. How far away was the second dog when the first dog reached
me?

(A) 15 m (B) 20 m (C) 30 m (D) 40 m (E) 50 m

1 2
9. The value of x2 + 2
when x = is closest to
x 3
(A) 0 (B) 1 (C) 2 (D) 3 (E) 4

10. A piece of paper in the shape of an equilateral triangle has one corner folded over,
as shown.
40◦

x◦

What is the value of x?

(A) 60 (B) 70 (C) 80 (D) 90 (E) 100

Questions 11 to 20, 4 marks each

11. Start with the number 1 and create the sequence

1, 2, 4, 8, 16, 22, 24, 28, . . .

where each number is the sum of the previous number and its final digit. How
many numbers in the sequence are less than 1000?

(A) 10 (B) 100 (C) 101 (D) 200 (E) 201


I3

12. A six-sided dice has the numbers 1, 2, 2, 3, 3 and 3 on its faces. Two such dice are
rolled and a score is made by adding the numbers on the uppermost faces. The
probability of rolling an odd score is
1 2 1 4 5
(A) (B) (C) (D) (E)
9 9 3 9 9

13. If x2 = x + 3, then x3 equals


(A) x + 6 (B) 2x + 6 (C) 3x + 9 (D) 4x + 3 (E) 27x + 9

14. The point T divides the side QR of the rectangle P QRS into two equal segments.
The point U divides P Q such that P U : U Q = 1 : 2. Point V divides SP such
that SV : V P = 1 : 3 and finally, point W divides RS such that RW : W S = 1 : 4.
Find the area of the quadrilateral T U V W if the area of P QRS equals 120.

S W R

P U Q

(A) 67 (B) 70 (C) 72 (D) 75 (E) 77

15. Three line segments of lengths 1, a and 2a are the sides of a triangle. Which of
the following defines all possible values of a?
1 1
(A) <a<1 (B) 0 < a < (C) a < 1 (D) for all a > 0 (E) for no a
3 3

16. The shaded segment in the circle below, centre O, has an area of 1 cm2 . The radius
of the circle, in centimetres, is

 
4 8 4 4 √
(A) (B) (C) (D) (E) 2 π
π π π−2 π
I4

17. Dan and Jane each have a measuring tape of length 1 m. Dan’s tape got stuck in
a door and was extended by 4 cm. Jane left her tape in a pocket and it shrank by
5 cm after washing. However, the centimetre marks on both tapes remained evenly
distributed.
Measuring the schoolyard, Dan noted the length as 23.75 m. What length will Jane
get measuring the same schoolyard with her tape?

(A) 23 m (B) 24 m (C) 25 m (D) 26 m (E) 27 m

18. In the regular hexagon pictured, the midpoints of the sides are joined to form the
shaded regular hexagon. What fraction of the larger hexagon is shaded?

3 2 5 1 7
(A) (B) (C) (D) (E)
4 3 6 2 8

19. A circular wheel of radius r rolls, without slipping, through half a revolution. The
point X is on the horizontal diameter at the start.

X X

The distance between the starting and finishing position of the point X is

(A) 2πr (B) (π + 2)r (C) (π − 2)r (D) 2(π + 1)r (E) 2(π − 1)r

20. The sport of bingbong involves two players. Each match consists of a number of
rounds and each round consists of a number of points. The first player to win four
points in a round wins the round. The first player to win six rounds in a match
wins the match.
Suppose that after a match of bingbong, the winner has won W points while the
loser has won L points. What is the largest possible value of L − W ?

(A) −6 (B) −4 (C) 0 (D) 12 (E) 14


I5

Questions 21 to 25, 5 marks each

21. In how many ways can the numbers 1, 2, 3, 4, 5, 6 be arranged in a row so that
the product of any two adjacent numbers is even?

(A) 64 (B) 72 (C) 120 (D) 144 (E) 720

22. Two circles, one of radius 1 and the other of radius 2, touch externally at P . A
straight line through P cuts the area formed by these two circles in the ratio 1 : 2.
In what ratio does this line cut the area of the smaller circle?

(A) 1 : 2 (B) 2 : 5 (C) 1 : 3 (D) 2 : 7 (E) 1 : 4

23. How many positive integers n are there such that 2n + 1 is a divisor of 8n + 46?

(A) 0 (B) 1 (C) 2 (D) 3 (E) 4

24. The rectangle P QRS shown has P Q = 4, P S = 12 and centre C. The two shaded
circles have radius 1 and touch P S at U and V where P U = 1 and P V = 4.
The line CW divides the unshaded area in half. The length of P W is

Q R

W
P U V S

2 2 1 1 1
(A) (B) (C) (D) (E)
7 5 4 3 2
I6

25. In 3013, King Warren of Australia is finally deposed. The five remaining earls argue
about which one of them will be king, and which one of the others will be treasurer.

Akaroa will be satisfied only if Darlinghurst or Erina is treasurer.


Bairnsdale will be satisfied only if Claremont is treasurer.
Claremont will be satisfied only if Darlinghurst is either king or treasurer.
Darlinghurst will be satisfied only if Akaroa is either king or treasurer.
Erina will be satisfied only if Akaroa is not king.

It is not possible for all five to be satisfied, so in the end they appoint king and
treasurer so that the other three earls are satisfied. Who becomes king?

(A) Akaroa (B) Bairnsdale (C) Claremont


(D) Darlinghurst (E) Erina

For questions 26 to 30, shade the answer as an integer from 0 to 999 in


the space provided on the answer sheet.

Question 26 is 6 marks, question 27 is 7 marks, question 28 is 8 marks,


question 29 is 9 marks and question 30 is 10 marks.

26. The 4-digit number pqrs has the property that pqrs × 4 = srqp. If p = 2, what is
the value of the 3-digit number qrs?

27. Three different non-zero digits are used to form six different 3-digit numbers. The
sum of five of them is 3231. What is the sixth number?

28. A hockey game between two teams is ‘relatively close’ if the number of goals scored
by the two teams never differ by more than two. In how many ways can the first
12 goals of a game be scored if the game is ‘relatively close’ ?

29. How many pairs (a, b) of positive integers are there such that a and b are factors
of 66 and a is a factor of b?

30. All the digits of the positive integer N are either 0 or 1. The remainder after
dividing N by 37 is 18. What is the smallest number of times that the digit 1 can
appear in N ?
Intermediate 2013 Answers
Question Answer
1 E

2 C

3 E

4 C

5 E

6 C

7 B

8 B

9 D

10 C

11 E

12 D

13 D

14 A

15 A

16 C

17 D

18 A

19 B

20 E

21 D

22 D

23 D

24 A

25 B

26 178

27 765

28 972

29 784

30 5
Australian Mathematics Competition
sponsored by the Commonwealth Bank
an activity of the australian mathematics trust

A u s t r a l i a n M at h e mat i c s T r u s t

NAME

YEAR TEACHER

2014
INTERMEDIATE DIVISION
AUSTRALIAN SCHOOL YEARS 9 and 10
TIME ALLOWED: 75 MINUTES

INSTRUCTIONS AND INFORMATION


GENERAL
1. Do not open the booklet until told to do so by your teacher.
2. NO calculators, maths stencils, mobile phones or other calculating aids are permitted. Scribbling paper, graph
paper, ruler and compasses are permitted, but are not essential.
3. Diagrams are NOT drawn to scale. They are intended only as aids.
4. There are 25 multiple-choice questions, each with 5 possible answers given and 5 questions that require a
whole number answer between 0 and 999. The questions generally get harder as you work through the paper.
There is no penalty for an incorrect response.
5. This is a competition not a test; do not expect to answer all questions. You are only competing against your
own year in your own country/Australian state so different years doing the same paper are not compared.
6. Read the instructions on the answer sheet carefully. Ensure your name, school name and school year are
entered. It is your responsibility to correctly code your answer sheet.
7. When your teacher gives the signal, begin working on the problems.

THE ANSWER SHEET


1. Use only lead pencil.
2. Record your answers on the reverse of the answer sheet (not on the question paper) by FULLY colouring the
circle matching your answer.
3. Your answer sheet will be scanned. The optical scanner will attempt to read all markings even if they are in
the wrong places, so please be careful not to doodle or write anything extra on the answer sheet. If you want
to change an answer or remove any marks, use a plastic eraser and be sure to remove all marks and smudges.

INTEGRITY OF THE COMPETITION


The AMT reserves the right to re-examine students before deciding whether to grant official status to their score.

©AMT Publishing 2014 amtt limited acn 083 950 341


Intermediate Division

Questions 1 to 10, 3 marks each

1. 1 + 2 − 3 − 4 + 5 + 6 − 7 − 8 + 9 + 10 =
(A) 0 (B) 1 (C) 10 (D) 11 (E) 19

2. In the diagram the value of x is


50◦
(A) 80 (B) 70 (C) 60
x◦ 120◦
(D) 50 (E) 40

3. If p = 9 and q = −3 then p2 − q 2 is equal to

(A) 64 (B) 72 (C) 84 (D) 90 (E) 96

4. What value can be placed in the shape to make this statement true?

2014 ÷ = 100

(A) 0.02014 (B) 0.2014 (C) 2.014 (D) 20.14 (E) 201.4

5 3
5. If of a number is 30, what is of the number?
6 4
(A) 22.5 (B) 24 (C) 25 (D) 27 (E) 40

6. This diagram is called an open square of order 4, since the


three sides are all the same length and each side has four
posts spaced evenly along it. The total number of posts
which would be evenly spaced along an open square of order
10 would be
(A) 26 (B) 27 (C) 28
(D) 30 (E) 32
I2

7. Which of the objects below is not the same as


the one on the right?

(A) (B) (C)

(D) (E)

8. This graph shows the number of goals scored by


Ranjit’s soccer team in each of the first thirteen 6
5
frequency

matches. After the fourteenth match, the me- 4


dian has increased but the mode has remained 3
the same. Which of the following best describes 2
1
the team’s score in this last game? 0
1 2 3 4 5
(A) Goals = 1 (B) Goals = 2 (C) Goals ≥ 2
number of goals
(D) Goals < 3 (E) Goals ≥ 3

9. Forty-eight pavers, each 1 m × 1 m in size, are used to form a path 1 metre wide
around a square garden. What is the area, in square metres, inside this path?

(A) 100 (B) 110 (C) 121 (D) 132 (E) 144

10. Each May a farmer plants barley seed and then in October he harvests 12 times the
weight of seed planted. From each harvest, he sells 50 tonnes and the rest he keeps as
seed for the next year’s crop. This year he has planted enough to harvest 120 tonnes.
How many tonnes did he plant last year?

(A) 5 (B) 10 (C) 20 (D) 30 (E) 60


I3

Questions 11 to 20, 4 marks each


11. If x is an integer and x < −1, which of the following expressions has the greatest
value?
1 1 1 1
(A) (B) (C) x + 1 (D) − (E) −
x x2 x2 x

12. The 11 boys in Tom’s cricket team have a contest to see how far they can throw a
cricket ball. Their results, to the nearest metre, are

19, 26, 31, 31, 31, 33, 37, 42, 42, 48, 56

Which of the following lists the statistical measures for these results in the correct
ascending order?

(A) mean, median, mode (B) median, mean, mode (C) mode, mean, median
(D) median, mode, mean (E) mode, median, mean

13. I have 800 mL of water in jug X and 800 mL of milk in jug Y . I pour 200 mL from jug
X into jug Y and stir the mixture thoroughly. I then pour 200 mL of the resulting
mixture from jug Y into jug X. What is the volume of milk that is now in jug X?

(A) 150 mL (B) 160 mL (C) 175 mL (D) 180 mL (E) 200 mL

14. The women’s world record for running 400 metres was set in Canberra at 47.60 sec-
onds. Which of the following is closest to the runner’s average speed, in kilometres
per hour?

(A) 22 (B) 24 (C) 26 (D) 28 (E) 30

15. Ten points Q, R, S, T , U , V , W , X, Y and Z are equally and consecutively spaced


on a circle. What is the size, in degrees, of the angle ∠QT W ?

(A) 36 (B) 54 (C) 60 (D) 72 (E) 75

16. A 3 by 5 grid of dots is set out as shown. How many


straight line segments can be drawn that join two of
these dots and pass through exactly one other dot?

(A) 14 (B) 20 (C) 22


(D) 24 (E) 30
I4

17. A hotel has rooms that can accommodate up to two people. Couples can share a
room, but otherwise men will share only with men and women only with women.
How many rooms are needed to guarantee that any group of 100 people can be
accommodated?
(A) 50 (B) 51 (C) 67 (D) 98 (E) 99

18. Two rectangular prisms are constructed. One measures 4 cm × 6 cm × x cm and the
other measures 3 cm × 8 cm × y cm where both x and y are integers. If they have
equal surface area, what is the smallest possible value of x + y?

(A) 11 (B) 21 (C) 26 (D) 42 (E) 63

19. A four-digit number abcd is called cool if a is divisible by 4, the two-digit number ab
is divisible by 5, the three-digit number abc is divisible by 6 and abcd is divisible by
7. How many cool numbers are there where 8 is not one of the digits?

(A) 3 (B) 4 (C) 5 (D) 6 (E) more than 6

20. The shape shown is formed from four identical arcs, each a
quarter of the circumference of a circle of radius 5 cm. What
is the area of the shape, in square centimetres?

(A) 100 − 20π (B) 100 (C) 25π + 25


(D) 25π (E) 100 − 25π

Questions 21 to 25, 5 marks each


21. Standard six-sided dice have their dots arranged so that the opposite faces add up
to 7. If 27 standard dice are arranged in a 3 × 3 × 3 cube on a solid table what is the
maximum number of dots that can be seen from one position?

(A) 90 (B) 94 (C) 153 (D) 154 (E) 189

22. There are 10 integers in a set. Some are odd and some are even. For each possible
pair selected from the set, the sum is written down. Of these 45 numbers, exactly 20
are even. How many of the numbers in the original set are even?

(A) 0 (B) 3 (C) 5 (D) 8 (E) 10


I5

2
23. Starting with of a tank of fuel, I set out to drive the 550 km from Scone to Canberra.
3 1
At Morisset, 165 km from Scone, I have of a tank remaining. If I continue with the
2
same fuel consumption per kilometre and without refuelling, what happens?
1
(A) I will arrive in Canberra with of a tank to spare.
9
1
(B) I will arrive in Canberra with of a tank to spare.
20
(C) I will run out of fuel precisely when I reach Canberra.
(D) I will run out of fuel 110 km from Canberra.
(E) I will run out of fuel 220 km from Canberra.

24. At a party, each person shakes hands with exactly three other people and no two
people shake hands with each other more than once. If fewer than fifteen handshakes
take place, what is the maximum number of people who can be at the party?
(A) 6 (B) 7 (C) 8 (D) 9 (E) 10

25. Thanom has a roll of paper consisting of a very long sheet of


thin paper tightly rolled around a cylindrical tube, forming
the shape indicated in the diagram.
Initially, the diameter of the roll is 12 cm and the diameter
of the tube is 4 cm. After Thanom uses half of the paper,
the diameter of the remaining roll is closest to
(A) 6 cm (B) 8 cm (C) 8.5 cm
(D) 9 cm (E) 9.5 cm

For questions 26 to 30, shade the answer as an integer from 0 to 999


in the space provided on the answer sheet.
Question 26 is 6 marks, question 27 is 7 marks, question 28 is 8 marks,
question 29 is 9 marks and question 30 is 10 marks.

26. In a 3 × 3 grid of points, many triangles can be formed using 3 of the points as
vertices. Three such triangles are shown below. Of all these possible triangles, how
many have all three sides of different lengths?
I6

27. Small squares of side x cm have been removed from


the corners, sides and centre of a square of side y cm
to form the gasket shown.
If x and y are prime numbers and the sum of the inside
and outside perimeters of the gasket, in centimetres, is
equal to the area of the gasket, in square centimetres,
what is the smallest possible value of the area of the
gasket?

28. In the diagram on the right, each circle has three or


six neighbours. Each circle will contain a number, and 0
each of the five missing numbers is the average of its
neighbours. What is the largest of the five missing
numbers?

1000

29. As shown in the diagram, you can create a grid


of squares 3 units high and 4 units wide using 31
matches. I would like to make a grid of squares
a units high and b units wide, where a < b are
positive integers. Determine the sum of the areas
of all such rectangles that can be made, each using
exactly 337 matches.

30. Consider the sequence a1 , a2 , a3 , a4 , . . . such that a1 = 2 and for every positive
integer n,

an+1 = an + pn , where pn is the largest prime factor of an .

The first few terms of the sequence are 2, 4, 6, 9, 12, 15, 20. What is the largest value
of n such that an is a four-digit number?
Intermediate 2014 Answers
Question Answer
1 D
2 B
3 B
4 D
5 D
6 C
7 A
8 E
9 C
10 A
11 E
12 E
13 B
14 E
15 D
16 C
17 B
18 B
19 A
20 E
21 C
22 C
23 A
24 C
25 D
26 40
27 88
28 625
29 704
30 198
Australian Mathematics Competition
sponsored by the Commonwealth Bank
an activity of the australian mathematics trust

A u s t r a l i a n M at h e mat i c s T r u s t

NAME

YEAR TEACHER

2015
INTERMEDIATE DIVISION
AUSTRALIAN SCHOOL YEARS 9 and 10
TIME ALLOWED: 75 MINUTES

INSTRUCTIONS AND INFORMATION


GENERAL
1. Do not open the booklet until told to do so by your teacher.
2. NO calculators, maths stencils, mobile phones or other calculating aids are permitted. Scribbling paper, graph
paper, ruler and compasses are permitted, but are not essential.
3. Diagrams are NOT drawn to scale. They are intended only as aids.
4. There are 25 multiple-choice questions, each with 5 possible answers given and 5 questions that require a
whole number answer between 0 and 999. The questions generally get harder as you work through the paper.
There is no penalty for an incorrect response.
5. This is a competition not a test; do not expect to answer all questions. You are only competing against your
own year in your own country/Australian state so different years doing the same paper are not compared.
6. Read the instructions on the answer sheet carefully. Ensure your name, school name and school year are
entered. It is your responsibility to correctly code your answer sheet.
7. When your teacher gives the signal, begin working on the problems.

THE ANSWER SHEET


1. Use only lead pencil.
2. Record your answers on the reverse of the answer sheet (not on the question paper) by FULLY colouring the
circle matching your answer.
3. Your answer sheet will be scanned. The optical scanner will attempt to read all markings even if they are in
the wrong places, so please be careful not to doodle or write anything extra on the answer sheet. If you want
to change an answer or remove any marks, use a plastic eraser and be sure to remove all marks and smudges.

INTEGRITY OF THE COMPETITION


The AMT reserves the right to re-examine students before deciding whether to grant official status to their score.

©AMT Publishing 2015 amtt limited acn 083 950 341


Intermediate Division

Questions 1 to 10, 3 marks each


1. What is the area of this triangle in square
centimetres? 2 cm
(A) 10 (B) 12 (C) 14 (D) 7 (E) 6
12 cm

2. A movie lasts for 2 1 hours. The movie is shown in two equal sessions.
3
For how many minutes does each session last?

(A) 85 (B) 70 (C) 80 (D) 65 (E) 75

3. If p = 11 and q = −4, then p2 − q 2 equals

(A) 105 (B) 137 (C) 117 (D) 115 (E) 94

4. 2015 − 20.15 equals

(A) 1984.85 (B) 1995.15 (C) 1994.85 (D) 1995.85 (E) 2035.15

5. What is the value of 2015 twenty-cent coins?

(A) $2015 (B) $107.50 (C) $17.50 (D) $403 (E) $43

6. Ana, Ben, Con, Dan and Eve are sitting around a ta-
ble in that order. Ana calls out the number 1, then Ana
Ben calls out the number 2, then Con calls out the
number 3, and so on. After a person calls out a num-
ber, the next person around the table calls out the Eve Ben
next number.
Anyone who calls out a multiple of 7 must immediately
leave the table.
Who is the last person remaining at the table? Dan Con

(A) Ana (B) Ben (C) Con


(D) Dan (E) Eve
I2

7. On a farm the ratio of horses to cows is 3 : 2 and ratio of cows to goats is 4 : 3.


The ratio of goats to horses is

(A) 5 : 7 (B) 3 : 8 (C) 3 : 5 (D) 5 : 18 (E) 1 : 2

8. Warren the window washer starts on the 38th floor of a building that has 12 windows
per floor. He washes all of the windows on each floor before moving down to the floor
below. Which floor is Warren on after he has washed 141 windows?
(A) 25th (B) 24th (C) 28th (D) 27th (E) 26th

9. A packet of lollies contains 5 blue lollies, 15 yellow lollies and some red lollies.
One-third of the lollies are red. What fraction of the lollies are yellow?
1 5 1 1 2
(A) (B) (C) (D) (E)
3 6 2 6 3

10. The diagram shows two small squares in opposite corners of


a large square. The squares have sides of length 1 cm, 2 cm
and 7 cm.
What is the area of the shaded pentagon?

(A) 18 cm2 (B) 16 cm2 (C) 22 cm2


(D) 24 cm2 (E) 20 cm2
I3

Questions 11 to 20, 4 marks each


11. Jenna measures three sides of a rectangle and gets a total of 80 cm. Dylan measures
three sides of the same rectangle and gets a total of 88 cm. What is the perimeter of
the rectangle?

(A) 112 cm (B) 132 cm (C) 96 cm (D) 168 cm (E) 156 cm

12. A bar-tailed godwit was recorded by satellite tag in 2007 to


have flown 11 500 km in eight days.
On average, approximately how many kilometres per hour
is that?
(A) 120 (B) 6 (C) 1 (D) 24 (E) 60

13. A cube has the letters A, C, M, T, H and S on its six faces. Here are two views of
this cube.
A
T

C AM
M

Which one of the following could be a third view of the same cube?

(A)
T (B) (C)
C (D) (E)
A T

C
M

M T S H S
H

C
A

14. Two ordinary dice are rolled. The two resulting numbers are multiplied together to
create a score. The probability of rolling a score that is a multiple of six is
1 5 1 1 1
(A) (B) (C) (D) (E)
6 12 4 3 2
I4

15. The diagram shows part of a tiled floor.


The tiles are all squares, of seven different
sizes. The smallest tile is 1 cm × 1 cm and
the next smallest is 3 cm × 3 cm.
What is the distance XY ?

(A) 346 cm (B) 19 cm Y
√ √
(C) 369 cm (D) 377 cm
(E) 20 cm X

16. How many three-digit numbers have no two digits the same?

(A) 764 (B) 900 (C) 648 (D) 520 (E) 480

17. The diagram shows two weightless balance beams.


Each tips as a result of an imbalance in the hanging
Y kg
weights. Each weight has mass X kg, Y kg or Z kg, as
shown. X kg
Y kg
Which of the following lists the weights from lightest
to heaviest?
Z kg
(A) X < Y < Z (B) X < Z < Y (C) Y < X < Z X kg
(D) Y < Z < X (E) Z < X < Y Y kg

18. A strip of paper 1 cm wide is folded 4 times to make a regular octagon as shown.

If the ends of the strip meet exactly when folded, how many centimetres long is the
strip?
√ √ √
(A) 8 2 (B) 8 (C) 4 + 4 2 (D) 16 (E) 16 − 4 2
I5

19. The country of Numismatica has six coins of the


following denominations: 1 cent, 2 cents, 4 cents, 4c
10c
10 cents, 20 cents and 40 cents.
Using the coins in my pocket, I can pay exactly for 2c
40c
any amount up to and including 200 cents.
20c 1c
What is the smallest number of coins I could have?
(A) 12 (B) 10 (C) 11 (D) 9 (E) 8

20. What fraction of the large triangle is shaded? 1


1 1 4 2
(A) (B) (C)
6 3 9 2
1 2 1
(D) (E)
2 5

2 1

Questions 21 to 25, 5 marks each


21. A student noticed that in a list of five integers, the mean, median and mode were
consecutive integers in ascending order. What is the largest range possible for these
five integers?

(A) 5 (B) 9 (C) 8 (D) 7 (E) 6

22. The square P QRS has sides of length 2 units and Q


P 2
J is the midpoint of P S. The line QJ intersects
the diagonal P R at L.
1
The length of LP is
L 2
√ √ √ J
2 3 2
(A) (B) (C) 1
3 3 2
√ √
2 3 2 2 S R
(D) (E)
3 3
I6

23. For each integer from 0 to 999, André wrote down the sum of its digits. What is the
average of the numbers that André wrote down?

(A) 13.5 (B) 15 (C) 12 (D) 12.5 (E) 10.5

24. Max’s journey around this grid starts on a grid point B C


on side AB.
He visits a grid point on each of sides BC, CD and
DA in order before returning to his starting point,
forming a quadrilateral.
Max does not visit corner points A, B, C or D.
How many journeys are possible which are not rect-
angles? (Note that a square is a rectangle.)
A D
(A) 256 (B) 252 (C) 64 (D) 248 (E) 76

25. It takes Nicolai one and a half hours to paint the walls of a room and two hours to
paint the ceiling. Elena needs exactly one hour to paint the walls of the same room
and one hour to paint the ceiling.
If Nicolai and Elena work together, what is the shortest possible time in minutes in
which they can paint the walls and the ceiling of the room?

(A) 72 (B) 60 (C) 83 (D) 75 (E) 76

For questions 26 to 30, shade the answer as an integer from 0 to 999


in the space provided on the answer sheet.
Question 26 is 6 marks, question 27 is 7 marks, question 28 is 8 marks,
question 29 is 9 marks and question 30 is 10 marks.

26. Mike has 2015 matches and uses them to build


a triangular pattern like the one shown, but as
big as possible. How many matches does he have
left over?
I7

1 1
27. How many positive integers n less than 2015 have the property that + can be
3 n
simplified to a fraction with denominator less than n?

28. A rectangle has all sides of integer length. When 3 units are added to the height
and 2 units to the width, the area of the rectangle is tripled. What is the sum of the
original areas of all such rectangles?

29. At Berracan station, northbound trains arrive every three minutes starting at noon
and finishing at midnight, while southbound trains arrive every five minutes starting
at noon and finishing at midnight. Each day, I walk to Berracan station at a random
time in the afternoon and wait for the first train in either direction. On average, how
many seconds should I expect to wait?

P
30. In a 14 × 18 rectangle ABCD, points P, Q, R and S A B
are chosen, one on each side of ABCD as pictured.
The lengths AP , P B, BQ, QC, CR, RD, DS and Q
SA are all positive integers and P QRS is a rectangle.
What is the largest possible area that P QRS could S
have?
D C
R
Intermediate 2015 Answers
Question Answer
1 B
2 B
3 A
4 C
5 D
6 D
7 E
8 D
9 C
10 C
11 A
12 E
13 A
14 B
15 E
16 C
17 E
18 C
19 B
20 B
21 D
22 E
23 A
24 D
25 A
26 37
27 224
28 44
29 74
30 150
Australian Mathematics Competition
sponsored by the Commonwealth Bank
an activity of the australian mathematics trust

A u s t r a l i a n M at h e mat i c s T r u s t

NAME

YEAR TEACHER

2016
INTERMEDIATE DIVISION
AUSTRALIAN SCHOOL YEARS 9 and 10
TIME ALLOWED: 75 MINUTES

INSTRUCTIONS AND INFORMATION


GENERAL
1. Do not open the booklet until told to do so by your teacher.
2. NO calculators, maths stencils, mobile phones or other calculating aids are permitted. Scribbling paper, graph
paper, ruler and compasses are permitted, but are not essential.
3. Diagrams are NOT drawn to scale. They are intended only as aids.
4. There are 25 multiple-choice questions, each with 5 possible answers given and 5 questions that require a
whole number answer between 0 and 999. The questions generally get harder as you work through the paper.
There is no penalty for an incorrect response.
5. This is a competition not a test; do not expect to answer all questions. You are only competing against your
own year in your own country/Australian state so different years doing the same paper are not compared.
6. Read the instructions on the answer sheet carefully. Ensure your name, school name and school year are
entered. It is your responsibility to correctly code your answer sheet.
7. When your teacher gives the signal, begin working on the problems.

THE ANSWER SHEET


1. Use only lead pencil.
2. Record your answers on the reverse of the answer sheet (not on the question paper) by FULLY colouring the
circle matching your answer.
3. Your answer sheet will be scanned. The optical scanner will attempt to read all markings even if they are in
the wrong places, so please be careful not to doodle or write anything extra on the answer sheet. If you want
to change an answer or remove any marks, use a plastic eraser and be sure to remove all marks and smudges.

INTEGRITY OF THE COMPETITION


The AMT reserves the right to re-examine students before deciding whether to grant official status to their score.

©AMT Publishing 2015 amtt limited acn 083 950 341


Questions – Intermediate Division

1. What is the value of 20 × 16?


(A) 320 (B) 140 (C) 2016 (D) 32 (E) 800

2. In the figure, the shaded region is what fraction of the


circle? 1
2
1 1 1
(A) (B) (C) 1
20 10 2
1
1 1 5
(D) (E) 4
60 40

3. The cycling road race through the Adelaide Hills started at 11:15 am and the winner
finished at 2:09 pm the same day. The winner’s time in minutes was

(A) 135 (B) 174 (C) 164 (D) 294 (E) 186

720163
4. The fraction is
2016

(A) between 0 and 1 (B) between 1 and 10 (C) between 10 and 100
(D) between 100 and 1000 (E) greater than 1000

5. What is the value of (1 ÷ 2) ÷ (3 ÷ 4) ?


2 3 3 1 1
(A) (B) (C) (D) (E)
3 2 8 6 24

6. 0.75% of a number is 6. The number is


(A) 800 (B) 300 (C) 1200 (D) 400 (E) 100

7. In the expression below, the letters A, B, C, D and E represent the numbers 1, 2, 3, 4


and 5 in some order.
A×B+C ×D+E
What is the largest possible value of the expression?

(A) 24 (B) 27 (C) 26 (D) 51 (E) 25

2016 AMC — Intermediate Questions 19


2016 AMC – Intermediate Questions 19
8. In each of these squares, the marked length is 1 unit. Which of the squares would
have the greatest perimeter?

1
1
P. Q. 1 R. S.
1

(A) P (B) Q (C) R (D) S (E) all are the same

9. On a clock face, a line is drawn between 9 and 3 and another 12


11 1
between 12 and 8. What is the acute angle between these
10 2
lines?
9 3
(A) 45◦ (B) 60◦ (C) 50◦
8 4
(D) 30◦ (E) 22.5◦
7 5
6

10. There are 3 blue pens, 4 red pens and 5 yellow pens in a box. Without looking, I
take pens from the box one by one. How many pens do I need to take from the box
to be certain that I have at least one pen of each colour?

(A) 8 (B) 9 (C) 10 (D) 11 (E) 12

11. In the diagram, the value of x is x◦


(A) 120 (B) 108 (C) 105 x◦
(D) 135 (E) 112.5 x◦

x◦

12. How far is it from A to B measured in a straight A


line? 4
√ 3
(A) 20 (B) 28 (C) 10 + 9 2
√ 4
(D) 8 + 9 2 (E) 16 4

5
5
3
B

20 2016 AMC — Intermediate Questions

20 2016 AMC – Intermediate Questions


13. A circle of radius 1 metre is inscribed inside a semicircle of
radius 2 metres. What is the area in square metres of the
semicircle not covered by the circle?

(A) 2π (B) π − 1 (C) 2


(D) 2π − 1 (E) π

14. The value of n for which 4n+1 = 210 is


(A) 9 (B) 8 (C) 4 (D) 10 (E) 2

15. Adrienne, Betty and Cathy were the only three competitors participating in a series
of athletic events. In each event, the winner gets 3 points, second gets 2 points and
third gets 1 point. After the events, Adrienne has 8 points, Betty has 11 points and
Cathy has 5 points. In how many events did Adrienne come second?

(A) 0 (B) 1 (C) 2 (D) 3 (E) 4

2016
16. What is the smallest number N for which is a perfect square?
N

(A) 14 (B) 2 (C) 56 (D) 12 (E) 7

17. Five people are sitting around a circle. Some always tell the truth, whilst the others
always lie. Each person claims to be sitting between two liars. How many of them
are telling the truth?

(A) 0 (B) 1 (C) 2 (D) 3 (E) 4

18. A cylindrical glass of (inside) diameter 6 cm and height 11 cm


is filled and then tilted to a 45◦ angle so that some water
overflows. How much water is left in it?
(A) 48π mL (B) 45π mL (C) 66π mL
(D) 72π mL (E) 63π mL

19. Ten students sit a test consisting of 20 questions. Two students get 8 questions correct
and one student gets 9 questions correct. The remaining seven students all get at
least 10 questions correct and the average number of questions answered correctly
by these seven students is an integer. If the average number of questions answered
correctly by all ten students is also an integer, then that integer is

(A) 10 (B) 11 (C) 12 (D) 13 (E) 14

2016 AMC — Intermediate Questions 21

2016 AMC – Intermediate Questions 21


20. This pedal-powered water pump is made 40
from bicycle parts.

PUMP
A 30-tooth gear on the pedals has a chain
to a 15-tooth gear. On the same axle as the
15-tooth gear is a 32-tooth gear that drives
a chain to a 40-tooth gear on the pump.
For every 100 complete revolutions of the 30 32
pedals, how many times does the gear on 15
the pump turn?

(A) 160 (B) 250 (C) 107


(D) 93 (E) 37 12

21. A gardener wishes to put a circular water feature (pool) in


a right-angled triangular plot that has sides of 6 m and 8 m
on its two smallest sides. What is the radius in metres of
the largest pool that will fit?
√ √
(A) 2 2 − 1 (B) 2 (C) 4 (D) 3 (E) 2 2

22. A sequence of 10 letters is made according to the following rules.

• The letter P can only be followed by Q or R.


• The letter Q can only be followed by R or S.
• The letter R can only be followed by S or T.
• The letter S can only be followed by T or P.
• The letter T can only be followed by P or Q.

How many possible sequences are there where the first, fourth, and tenth letters are
all Q?

(A) 63 (B) 39 (C) 32 (D) 45 (E) 36

23. Cynthia’s afternoon train normally arrives at her station at 5:30 pm each day, where
she is picked up by Alan and driven home.
One day she was on an earlier train which arrived at 5 pm, and she decided to walk in
the direction Alan was coming from home. Alan had left in time to meet the 5:30 pm
train, but this time he picked up Cynthia and they arrived home 10 minutes earlier
than usual.
For how many minutes had Cynthia walked before Alan picked her up?

(A) 20 (B) 30 (C) 25 (D) 10 (E) 15

22 2016 AMC — Intermediate Questions

22 2016 AMC – Intermediate Questions


24. You have an unlimited supply of five different coloured
pop-sticks, and want to make as many different coloured
equilateral triangles as possible, using three sticks.

r ed
r ed
One example is shown here.
Two triangles are not considered different if they are
rotations or reflections of each other.
How many different triangles are possible?
blue
(A) 35 (B) 5 (C) 20 (D) 56 (E) 10

25. A super-Fibonacci sequence is a list of whole numbers with the property that, from
the third term onwards, every term is the sum of all of the previous terms. For
example,
1, 4, 5, 10, . . .
How many super-Fibonacci sequences starting with 1 involve the number 2016?

(A) 1 (B) 3 (C) 5 (D) 7 (E) 9

26. The right-angled triangle ABC has area 2016 cm2 . A


Lines AI, IF , IE, EH, HD, DG divide the large
triangle into seven smaller triangles of equal area. F
If BIE is an isosceles triangle, find the length of BG
in centimetres. E

B C
G H I

27. A symmetrical cross with equal arms has an area of 2016 cm2 and all sides of in-
teger length in centimetres. What is the smallest perimeter the cross can have, in
centimetres?

2016 AMC — Intermediate Questions 23

2016 AMC – Intermediate Questions 23


28. The ten students in Malcolm’s maths class all took a test. The scores of the other
nine students were 82, 83, 85, 89, 90, 92, 95, 97, and 98, and Malcolm’s score was a
whole number.
The teacher had made a mistake in calculating Malcolm’s score. After she corrected
her mistake, both the mean and the median of all the scores increased by 0.5.
There are two possible correct scores that Malcolm could have had. What is the sum
of these two scores?

29. A high school marching band can be arranged in a rectangular formation with exactly
three boys in each row and exactly five girls in each column. There are several sizes
of marching band for which this is possible. What is the sum of all such possible
sizes?

30. Around a circle, I place 64 equally


spaced points, so that there are
64 × 63 ÷ 2 = 2016 possible chords
between these points.
I draw some of these chords, but
each chord cannot cut across more
than one other chord.
What is the maximum number of
chords I can draw?

24 2016 AMC — Intermediate Questions

24 2016 AMC – Intermediate Questions


Australian Mathematics Competition
sponsored by the Commonwealth Bank
an activity of the australian mathematics trust

A u s t r a l i a n M at h e mat i c s T r u s t

NAME

YEAR TEACHER

2017
INTERMEDIATE DIVISION
AUSTRALIAN SCHOOL YEARS 9 and 10
TIME ALLOWED: 75 MINUTES

INSTRUCTIONS AND INFORMATION


GENERAL
1. Do not open the booklet until told to do so by your teacher.
2. NO calculators, maths stencils, mobile phones or other calculating aids are permitted. Scribbling paper, graph
paper, ruler and compasses are permitted, but are not essential.
3. Diagrams are NOT drawn to scale. They are intended only as aids.
4. There are 25 multiple-choice questions, each with 5 possible answers given and 5 questions that require a
whole number answer between 0 and 999. The questions generally get harder as you work through the paper.
There is no penalty for an incorrect response.
5. This is a competition not a test; do not expect to answer all questions. You are only competing against your
own year in your own country/Australian state so different years doing the same paper are not compared.
6. Read the instructions on the answer sheet carefully. Ensure your name, school name and school year are
entered. It is your responsibility to correctly code your answer sheet.
7. When your teacher gives the signal, begin working on the problems.

THE ANSWER SHEET


1. Use only lead pencil.
2. Record your answers on the reverse of the answer sheet (not on the question paper) by FULLY colouring the
circle matching your answer.
3. Your answer sheet will be scanned. The optical scanner will attempt to read all markings even if they are in
the wrong places, so please be careful not to doodle or write anything extra on the answer sheet. If you want
to change an answer or remove any marks, use a plastic eraser and be sure to remove all marks and smudges.

INTEGRITY OF THE COMPETITION


The AMT reserves the right to re-examine students before deciding whether to grant official status to their score.

©AMT Publishing 2015 amtt limited acn 083 950 341


Intermediate2017 Answers
Question Answer
1 C
2 A
3 C
4 C
5 C
6 E
7 B
8 A
9 A
10 D
11 E
12 C
13 E
14 D
15 D
16 B
17 B
18 E
19 D
20 E
21 C
22 A
23 D
24 E
25 E
26 807
27 648
28 56
29 286
30 99
Australian Mathematics Competition
sponsored by the Commonwealth Bank
an activity of the australian mathematics trust

A u s t r a l i a n M at h e mat i c s T r u s t

NAME

YEAR TEACHER

2018
INTERMEDIATE DIVISION
AUSTRALIAN SCHOOL YEARS 9 and 10
TIME ALLOWED: 75 MINUTES

INSTRUCTIONS AND INFORMATION


GENERAL
1. Do not open the booklet until told to do so by your teacher.
2. NO calculators, maths stencils, mobile phones or other calculating aids are permitted. Scribbling paper, graph
paper, ruler and compasses are permitted, but are not essential.
3. Diagrams are NOT drawn to scale. They are intended only as aids.
4. There are 25 multiple-choice questions, each with 5 possible answers given and 5 questions that require a
whole number answer between 0 and 999. The questions generally get harder as you work through the paper.
There is no penalty for an incorrect response.
5. This is a competition not a test; do not expect to answer all questions. You are only competing against your
own year in your own country/Australian state so different years doing the same paper are not compared.
6. Read the instructions on the answer sheet carefully. Ensure your name, school name and school year are
entered. It is your responsibility to correctly code your answer sheet.
7. When your teacher gives the signal, begin working on the problems.

THE ANSWER SHEET


1. Use only lead pencil.
2. Record your answers on the reverse of the answer sheet (not on the question paper) by FULLY colouring the
circle matching your answer.
3. Your answer sheet will be scanned. The optical scanner will attempt to read all markings even if they are in
the wrong places, so please be careful not to doodle or write anything extra on the answer sheet. If you want
to change an answer or remove any marks, use a plastic eraser and be sure to remove all marks and smudges.

INTEGRITY OF THE COMPETITION


The AMT reserves the right to re-examine students before deciding whether to grant official status to their score.

©AMT Publishing 2015 amtt limited acn 083 950 341


2018 AMC
Intermediate Questions

Questions – Intermediate Division


2018 − 18
1. The value of is
1000
(A) 0.02 (B) 0.1 (C) 1 (D) 2 (E) 2000

2. What value is indicated on this charisma-meter?


(A) 36.65 (B) 37.65 (C) 38.65 36 38
(D) 37.15 (E) 37.3

3. What is the difference between the sum and the product of 4 and 5?
(A) 1 (B) 8 (C) 9 (D) 11 (E) 20

4. In the diagram, P QRS is a square. What is the size of ∠XP Y ? P Q


25◦
(A) 25◦ (B) 30◦ (C) 35◦
X
(D) 40◦ (E) 45◦ ◦
25

S R
Y

5. Which of the following is not a whole number?


(A) 350 ÷ 2 (B) 350 ÷ 7 (C) 350 ÷ 5 (D) 350 ÷ 25 (E) 350 ÷ 20

6. Nora, Anne, Warren and Andrew bought plastic capital


letters to spell each of their names on their birthday cakes. A NNE
Their birthdays are on different dates, so they planned to
reuse letters on different cakes.
What is the smallest number of letters they needed?
(A) 8 (B) 9 (C) 10
(D) 11 (E) 12

7. In years, 2018 days is closest to


(A) 4.5 years (B) 5 years (C) 5.5 years (D) 6 years (E) 6.5 years

c Australian Mathematics Trust www.amt.edu.au 22


2018 AMC
Intermediate Questions

8. Two paths from A to C are pictured. C


The stepped path consists of horizontal and vertical
segments, whereas the dashed path is straight.
What is the difference in length between the two
3m
paths?
(A) 1 m (B) 2 m (C) 3 m
(D) 4 m (E) 0 m A B
4m

9. The value of 9 × 1.2345 − 9 × 0.1234 is


(A) 9.9999 (B) 9 (C) 9.0909 (D) 10.909 (E) 11.1111

10. What fraction of this regular hexagon is shaded?


1 2 3 3 4
(A) (B) (C) (D) (E)
2 3 4 5 5

11. The cost of feeding four dogs for three days is $60. Using the same food costs per dog per
day, what would be the cost of feeding seven dogs for seven days?
(A) $140 (B) $200 (C) $245 (D) $350 (E) $420

12. In a certain year there were exactly four Tuesdays and exactly four Fridays in the month
of December. What day of the week was 31 December?
(A) Monday (B) Wednesday (C) Thursday (D) Friday (E) Saturday

13. Fill in this diagram so that each of the rows, columns and diago-
nals adds to 18.
What is the sum of all the corner numbers?
6
(A) 20 (B) 22 (C) 23
(D) 24 (E) 25 4

14. The sum of 4 consecutive integers is t.


In terms of t, the smallest of the four integers is
t − 10 t−2 t−3 t−4 t−6
(A) (B) (C) (D) (E)
4 4 4 4 4

c Australian Mathematics Trust www.amt.edu.au 23


2018 AMC
Intermediate Questions

15. A 3-dimensional object is formed by gluing six identical cubes together. Four of the
diagrams below show this object viewed from different angles, but one diagram shows a
different object. Which diagram shows the different object?

(A) (B) (C) (D) (E)

16. In the circle shown, C is the centre and A, B, D and E B


all lie on the circumference.
Reflex ∠BCD = 200◦ , ∠DCA = x◦ and ∠BCA = 3x◦
as shown.
The ratio of ∠DAC : ∠BAC is A 3x◦
(A) 3 : 1 (B) 5 : 2 (C) 8 : 3 C
x◦
(D) 7 : 4 (E) 7 : 3 200◦
E
D

17. Allan and Zarwa are playing a game tossing a coin. Allan wins as soon as a head is tossed
and Zarwa wins if two tails are tossed. The probability that Allan wins is
1 3 5 2 3
(A) (B) (C) (D) (E)
2 5 8 3 4

18. In this expression


1 1 1 1 1
3 4 5 6 7
we place either a plus sign or a minus sign in each box so that the result is the smallest
positive number possible. The result is
1 1 1
(A) between 0 and (B) between and
100 100 50
1 1 1 1
(C) between and (D) between and
50 20 20 10
1
(E) between and 1
10

c Australian Mathematics Trust www.amt.edu.au 24


2018 AMC
Intermediate Questions

19. A town is laid out in a square of side 1 kilometre, with six straight roads
as shown.
Each day the postman must walk the full length of every road at least
once, starting wherever he likes and ending wherever he likes.
How long is the shortest route he can take, in kilometres?

2 √ √ √ √
(A) 4 + (B) 4 + 2 (C) 4 + 2 2 (D) 4 + 3 2 (E) 5 + 2 2
2

20. A rectangle with integer sides has a diagonal stripe 1


which starts 1 unit from the diagonal corners, as
in the diagram. 1
The area of the stripe is exactly half of the area of
the rectangle.
What is the perimeter of this rectangle?
(A) 14 (B) 16 (C) 18
(D) 20 (E) 22
1

21. How many digits does the number 2018 have?


(A) 24 (B) 38 (C) 18 (D) 36 (E) 25

22. In this subtraction, the first number has 100 digits and the second number has 50 digits.

111 . . . . 111} − 222


| . . {z | .{z
. . 222}
100 digits 50 digits

What is the sum of the digits in the result?


(A) 375 (B) 420 (C) 429 (D) 450 (E) 475

23. Suppose p is a two-digit number and q has the same digits, but in reverse order. The
number p2 − q 2 is a non-zero perfect square. The sum of the digits of p is
(A) 7 (B) 9 (C) 11 (D) 12 (E) 13

c Australian Mathematics Trust www.amt.edu.au 25


2018 AMC
Intermediate Questions

Q
24. In triangle 4P QR, U is a point on P R, S
is a point on P Q, T is a point on QR with
U S k RQ, and U T k P Q.
The area of 4P SU is 120 cm2 and the area of T

>>
4T U R is 270 cm2 .
The area of 4QST , in square centimetres, is S

<
(A) 150 (B) 160 (C) 170 >>
270

<
(D) 180 (E) 200 120
P U R

25. This year Ann’s age is the sum of the digits of her maths teacher’s age. In five years Ann’s
age will be the product of the digits of her maths teacher’s age at that time.
How old is Ann now?
(A) 11 (B) 13 (C) 15 (D) 14 (E) 16

26. I have a three-digit number, and I add its digits to create its digit sum. When the digit
sum of my number is subtracted from my number, the result is the square of the digit sum.
What is my three-digit number?

27. A road from Tamworth to Broken Hill is 999 km long. There are road signs each kilometre
along the road that show the distances (in kilometres) to both towns as shown in the
diagram.

0|999 1|998 2|997 3|996 ··· 998|1 999|0

How many road signs are there that use exactly two different digits?

28. In the division shown, X, Y and Z are different non-zero digits.

Z X rem. Y
8 X Y Z

What is the three-digit number XY Z?

29. An infinite increasing list of numbers has the property that the median of the first n terms
equals the nth odd positive integer. How many numbers in the list are less than 2018?

c Australian Mathematics Trust www.amt.edu.au 26


2018 AMC
Intermediate Questions

30. For n ≥ 3, a pattern can be made by overlapping n circles, each


of circumference 1 unit, so that each circle passes through a
central point and the resulting pattern has order-n rotational
symmetry.
For instance, the diagram shows the pattern where n = 7.
If the total length of visible arcs is 60 units, what is n?

c Australian Mathematics Trust www.amt.edu.au 27


Intermediate 2018 Answers
Question Answer
1 D
2 E
3 D
4 D
5 E
6 B
7 C
8 B
9 A
10 B
11 C
12 A
13 D
14 E
15 A
16 E
17 E
18 C
19 E
20 A
21 A
22 D
23 C
24 D
25 B
26 156
27 40
28 435
29 505
30 118
AUSTRALIAN MATHEMATICS
COMPETITION
Intermediate Years 9 & 10
(Australian school years)

THURSDAY 1 AUGUST 2019

NAME:

TIME ALLOWED: 75 minutes

INSTRUCTIONS AND INFORMATION

General
1 Do not open the booklet until told to do so by your teacher.
2 NO calculators, maths stencils, mobile phones or other calculating aids are permitted.
Scribbling paper, graph paper, ruler and compasses are permitted, but are not essential.
3 Diagrams are NOT drawn to scale. They are intended only as aids.
4 There are 25 multiple-choice questions, each requiring a single answer, and 5 questions that
require a whole number answer between 0 and 999. The questions generally get harder as
you work through the paper. There is no penalty for an incorrect response.
5 This is a competition not a test; do not expect to answer all questions. You are only
competing against your own year in your own country/Australian state so different years
doing the same paper are not compared.
6 Read the instructions on the answer sheet carefully. Ensure your name, school name and
school year are entered. It is your responsibility to correctly code your answer sheet.
7 When your teacher gives the signal, begin working on the problems.

The answer sheet


1 Use only lead pencil.
2 Record your answers on the reverse of the answer sheet (not on the question paper) by
FULLY colouring the circle matching your answer.
3 Your answer sheet will be scanned. The optical scanner will attempt to read all markings
even if they are in the wrong places, so please be careful not to doodle or write anything
extra on the answer sheet. If you want to change an answer or remove any marks, use a
plastic eraser and be sure to remove all marks and smudges.

Integrity of the competition


The AMT reserves the right to re-examine students before deciding whether to grant official
status to their score.
Reminder: You may sit this competition once, in one division only, or risk no score.

Copyright © 2019 Australian Mathematics Trust AMTT Limited ACN 083 950 341
2019 AMC — Intermediate
Intermediate Division

Questions 1 to 10, 3 marks each


1. The value of 20.19 − 19 is
(A) 39.19 (B) 20.38 (C) 20 (D) 1.19 (E) 1

2. Sharyn’s piano lesson was 40 minutes long, and


finished at 4.10 pm. When did it start?
(A) 3.30 pm (B) 3.40 pm (C) 3.50 pm
(D) 4.40 pm (E) 4.50 pm

3. Which of the following is closest to 7 × 1.8?


(A) 10 (B) 11 (C) 12 (D) 13 (E) 14

4. What is the area of the shaded triangle?


(A) 8 m2 (B) 12 m2 (C) 14 m2 4m
(D) 20 m2 (E) 24 m2
6m 4m

5. Five-eighths of a number is 200. What is the number?


(A) 120 (B) 320 (C) 275 (D) 75 (E) 280

6. Every row and every column of this 3 × 3 square must contain 1


each of the numbers 1, 2 and 3.
2 N
What is the value of N + M ?
(A) 2 (B) 3 (C) 4 (D) 5 (E) 6 M
I22

2019 AMC — Intermediate


7. A piece of paper is folded in three, then a semi-circular cut and a straight cut are
made, as shown in the diagram.

 

When the paper is unfolded, what does it look like?


(A) (B) (C)

(D) (E)

8. When a rectangle is cut in half, two squares are formed. If each square has a perimeter
of 48, what is the perimeter of the original rectangle?
(A) 96 (B) 72 (C) 36 (D) 24 (E) 12

9. Consider the undulating number sequence

1, 4, 7, 4, 1, 4, 7, 4, 1, 4, . . . ,

which repeats every four terms. The running total of the first 3 terms is 12. The
running total of the first 7 terms is 28.
Which one of the following is also a running total of this sequence?
(A) 61 (B) 62 (C) 67 (D) 66 (E) 65

10. Sebastien has a Personal Identification Number (PIN) consisting of four digits. The
first three digits in order are 591. If Sebastien’s PIN is divisible by 3, then how many
possibilities are there for the final digit?
(A) 1 (B) 2 (C) 3 (D) 4 (E) 5
I33

2019 AMC — Intermediate


Questions 11 to 20, 4 marks each
11. A quadrilateral ABCD has ADBC, AB = BC and AC = CD. The external
∠CDE = 140◦ . What is the value, in degrees, of ∠ABC?

B C
>

140◦
>
A D E

(A) 90 (B) 100 (C) 110 (D) 120 (E) 130

12. In my dance class, 14 students are taller than Bob, and 12 are shorter than Alice.
Four students are both shorter than Alice and taller than Bob. How many students
are in my dance class?
(A) 22 (B) 24 (C) 26 (D) 28 (E) 30

13. Three equilateral triangles are joined to form the D X C


quadrilateral ABCD shown.
The point X is halfway along CD.
What fraction of the area of ABCD is shaded?
1 3 2 5 3
(A) (B) (C) (D) (E) A B
2 4 3 6 5

14. In a year 10 Maths class, there are 30 students. Each student is either 15 or 16 years
old, and either left- or right-handed.
The ratio of right-handed students to left-handed students is 4 : 1, the ratio of 15 year
olds to 16 year olds is 1 : 2 and the ratio of left-handed 15 year olds to left-handed
16 year olds is 1 : 5.
If the names of the students in this class are placed in a hat and one is selected
at random, what is the probability that the student selected is 15 years old and
right-handed?
1 1 3 1 4
(A) (B) (C) (D) (E)
30 6 10 2 5
I44

2019 AMC — Intermediate


15. The set of four rollers shown has fixed axles
and transfers rotation from each roller to
the next without slipping. Their diameters 32
are 21 cm, 32 cm, 50 cm and 14 cm respec- 14
tively. 21
While the 21 cm roller makes a full rotation 50
(360◦ ), through which angle does the 14 cm
roller rotate?
(A) 180◦ (B) 310◦ (C) 360◦
(D) 540◦ (E) 620◦

3
16. In a box of apples, of the apples are red and the rest are green. Five more green
7
5
apples are added to the box. Now of the apples are green.
8
How many apples are there now in the box?
(A) 32 (B) 33 (C) 38 (D) 40 (E) 48

17. Asha chooses a whole number from 1 to 5 and announces it. Then Richy chooses a
whole number from 1 to 5 and announces it. Finally, Asha chooses a whole number
from 1 to 5 and announces it.
If the sum of the three numbers announced is a multiple of 7, then Asha wins;
otherwise, Richy wins.
What number should Asha choose on her first turn to guarantee that she can win?
(A) 1 (B) 2 (C) 3 (D) 4 (E) 5
I55

2019 AMC — Intermediate


18. The area of the shaded triangle inside this rect-
angle is
1
(A) (x + y)2 (B) x(x + y) (C) y(x + y) y
2
x
1 1
(D) (y 2 − x2 ) (E) (x2 + y 2 )
2 2
x y

19. These three cubes are labelled in exactly the same way, with the 6 letters A, M, C,
D, E and F on their 6 faces:
C

M
F

M E D C
F

The cubes are now placed in a row so that the front looks like this:

A M CF
When we look at the cubes from the opposite side, we will see

(A) (B) (C)


A M E M E A
M
A
E

(D) (E)
A M M A E
E

20. Five numbers are placed in a row. From the third number on, each number is the
average of the previous two numbers. The first number is 12 and the last number
is 7. What is the third number?
(A) 4 (B) 5 (C) 6 (D) 7 (E) 8
I66

2019 AMC — Intermediate


Questions 21 to 25, 5 marks each
21. A pool can be filled through three pipes that can be used together or separately.
If only the first pipe is used, the pool is filled in 21 hours. If only the second pipe is
used, the pool is filled in 24 hours. If all three pipes are used, the pool is filled in 8
hours.
How long will it take to fill the pool using only the third pipe?
(A) 12 hours (B) 14 hours (C) 27 hours (D) 28 hours (E) 30 hours

22. A 4 cm × 4 cm board can have 1 cm3 cubes placed


on it as shown.
The board is cleared, then a number of these cubes
are placed on the grid. The front and right side
views are shown.
What is the maximum number of cubes there could
FRO S IDE
be on the board? NT HT
(A) 10 (B) 11 (C) 16 (D) 17 (E) 18 RIG

FRONT RIGHT SIDE

23. Manny has three ways to travel the 8 kilometres from home to work: driving his car
takes 12 minutes, riding his bike takes 24 minutes and walking takes 1 hour and 44
minutes. He wants to know how to get to work as quickly as possible in the event
that he is riding his bike and gets a flat tyre.
He has three strategies:
(i) If he is close to home, walk back home and then drive his car.
(ii) If he is close to work, just walk the rest of the way.
(iii) For some intermediate distances, spend 20 minutes fixing the tyre and then
continue riding his bike.
He knows there are two locations along the route to work where the strategy should
change. How far apart are they?
(A) 2 km (B) 3 km (C) 4 km (D) 5 km (E) 6 km
I77

2019 AMC — Intermediate


24. Out of modern musical theory comes the following question.
Twelve points are equally spaced around a circle. Three
points are to be joined to make a triangle.
We count two triangles as being the same only if they match
perfectly after rotating, but not reflecting. For instance, the
two triangles shown are the same.
How many different triangles can be made?
(A) 10 (B) 14 (C) 19 (D) 20 (E) 22

25. A circular coin of radius 1 cm rolls around the inside of a


square without slipping, always touching the boundary of
the square.
When it returns to where it started, the coin has performed
exactly one whole revolution.
In centimetres, what is the side length of the square?
π
(A) π (B) 3.5 (C) 1 + π (D) 4 (E) 2 +
2

For questions 26 to 30, shade the answer as an integer from 0 to 999


in the space provided on the answer sheet.
Questions 26–30 are worth 6, 7, 8, 9 and 10 marks, respectively.

26. A positive whole number is called stable if at least one of its digits has the same value
as its position in the number. For example, 78247 is stable because a 4 appears in
the 4th position. How many stable 3-digit numbers are there?

27. When I divide an integer by 15, the remainder is an integer from 0 to 14. When I
divide an integer by 27, the remainder is an integer from 0 to 26.
For instance, if the integer is 100 then the remainders are 10 and 19, which are
different.
How many integers from 1 to 1000 leave the same remainders after division by 15
and after division by 27?
I88

2019 AMC — Intermediate


28. The number 35 has the property that when its digits are both increased by 2, and
then multiplied, the result is 5 × 7 = 35, equal to the original number.
Find the sum of all two-digit numbers such that when you increase both digits by 2,
and then multiply these numbers, the product is equal to the original number.

29. The Leader of Zip decrees that the digit 0, since it represents nothing, will no longer
be used in any counting number. Only counting numbers without 0 digits are allowed.
So the counting numbers in Zip begin 1, 2, 3, 4, 5, 6, 7, 8, 9, 11, 12, . . . , where the
tenth counting number is 11.
When you write out the first one thousand allowable counting numbers in Zip, what
are the last three digits of the final number?

30. Antony the ant is at the top-left corner


of this brick wall and needs to get to the start
bottom-right corner.
Because it is a hot day, he avoids the
dark bricks and only walks on the cooler
white mortar between the bricks and at
the top and bottom of the wall.
There are 18 rows of bricks, each with
7 whole bricks and one half-brick in an
alternating pattern.
How many different ways are there for
Antony to walk to the opposite corner
as quickly as possible?
finish
Intermediate 2019 Answers
Question Answer
1 D
2 A
3 D
4 B
5 B
6 C
7 B
8 B
9 E
10 D
11 B
12 A
13 C
14 C
15 D
16 D
17 A
18 E
19 D
20 E
21 D
22 D
23 C
24 C
25 E
26 252
27 119
28 127
29 331
30 103
2020 AMC
AUSTRALIAN MATHEMATICS COMPETITION

Intermediate Years 9–10


(Australian school years)

THURSDAY 30 JULY 2020

NAME

TIME ALLOWED: 75 MINUTES

INSTRUCTIONS AND INFORMATION

General
1. Do not open the booklet until told to do so by your teacher.
2. NO calculators, maths stencils, mobile phones or other calculating aids are permitted.
Scribbling paper, graph paper, ruler and compasses are permitted, but are not essential.
3. Diagrams are NOT drawn to scale. They are intended only as aids.
4. There are 25 multiple-choice questions, each requiring a single answer, and 5 questions that
require a whole number answer between 0 and 999. The questions generally get harder as
you work through the paper. There is no penalty for an incorrect response.
5. This is a competition not a test; do not expect to answer all questions. You are only
competing against your own year in your own country/Australian state so different years
doing the same paper are not compared.
6. Read the instructions on the answer sheet carefully. Ensure your name, school name and
school year are entered. It is your responsibility to correctly code your answer sheet.
7. When your teacher gives the signal, begin working on the problems.

The answer sheet


1. Use only lead pencil.
2. Record your answers on the reverse of the answer sheet (not on the question paper) by
FULLY colouring the circle matching your answer.
3. Your answer sheet will be scanned. The optical scanner will attempt to read all markings
even if they are in the wrong places, so please be careful not to doodle or write anything
extra on the answer sheet. If you want to change an answer or remove any marks, use a
plastic eraser and be sure to remove all marks and smudges.

Integrity of the competition


The AMT reserves the right to re-examine students before deciding whether to grant official
status to their score.
Reminder: You may sit this competition once, in one division only, or risk no score.

Copyright © 2020 Australian Mathematics Trust


ACN 083 950 341
Intermediate Division
Questions 1 to 10, 3 marks each

1. 2 − (0 − (2 − 0)) =
(A) −4 (B) −2 (C) 0 (D) 2 (E) 4

2. 1000% of 2 is equal to
(A) 0.002 (B) 20 (C) 200 (D) 1002 (E) 2000

3. In the diagram provided, find the sum of x and y.


(A) 30 (B) 75 (C) 95 y◦

(D) 105 (E) 180

|
x◦
105◦

1+2+3+4+5 1+2
4. − =
1+2+3+4 1+2+3
5 7
(A) 3 (B) (C) 1 (D) (E) 2
6 6

5. Sebastien is thinking of two numbers whose sum is 26 and whose difference is 14.
The product of Sebastien’s two numbers is
(A) 80 (B) 96 (C) 105 (D) 120 (E) 132

6. Which of the shapes in the diagram have


equal area? P R

(A) All of the shapes have equal area.


(B) Only Q and S have equal area.
(C) Only R and T have equal area. S
(D) Only P, R and T have equal area. Q
(E) P, R and T have equal area, and Q
and S have equal area. T

7. 123456 − 12345 + 1234 − 123 + 12 − 1 =

(A) 33333 (B) 101010 (C) 111111 (D) 122223 (E) 112233
I 22020 Australian Mathematics Competition — Intermediate

4 5  7
8. If of of of is equal to 1, then the value of  is
5 6 7 8
(A) 6 (B) 8 (C) 10 (D) 12 (E) 14

9. A piece of paper is folded twice as shown and cut along the dotted lines.



Once unfolded, which letter does the piece of paper most resemble?
(A) M (B) O (C) N (D) B (E) V

10. An equilateral triangle is subdivided into a number of


smaller equilateral triangles, as shown. The shaded
triangle has side length 2. What is the perimeter of
the large triangle?
(A) 24 (B) 27 (C) 30
(D) 33 (E) 36

Questions 11 to 20, 4 marks each

11. Triangle XY S is enclosed by rectangle Q


P 6 cm X 10 cm
P QRS as shown in the diagram.
In square centimetres, what is the area
of triangle XY S?
8 cm
(A) 82 (B) 88 (C) 94
(D) 112 (E) 130
Y
6 cm

S R

1 1 1 1 1 1 1 1
12. Let X = 1 − + − + − and Y = 1 − + − . Then X − Y is equal to
3 5 7 9 11 3 5 7
2 1 1 1 2
(A) (B) (C) (D) (E)
99 11 10 2 9
2020 Australian Mathematics Competition — IntermediateI 3

13. The number 25 can be written as the sum of three different primes less than 20. For
instance, 25 = 5 + 7 + 13.
How many multiples of 10 can be written as the sum of three different primes less
than 20?
(A) 1 (B) 2 (C) 3 (D) 4 (E) 5

14. In this circle with centre O, four triangles are


drawn, with angles as shown.
What is the value of x?
(A) 10 (B) 15 (C) 18 2x◦ x◦
O
(D) 24 (E) 36

3x◦

4x

15. There are 10 children in a classroom. The ratio of boys to girls increases when another
girl and another boy enter the room. What is the greatest number of boys that could
have been in the room at the beginning?
(A) 1 (B) 4 (C) 5 (D) 6 (E) 9

16. Two triangles, A and B, have the same area. Triangle A is isosceles and triangle B
is right-angled.

5 cm 5 cm
A B

6 cm 6 cm

The difference between the perimeters of triangle A and triangle B is


(A) nothing (B) between 0 cm and 1 cm (C) between 1 cm and 2 cm
(D) between 2 cm and 3 cm (E) more than 3 cm

17. A list of numbers has first term 2 and second term 5. The third term, and each term
after this, is found by multiplying the two preceding terms together:

2, 5, 10, 50, 500, . . .

The value of the eighth term is


(A) 25 × 58 (B) 28 × 59 (C) 28 × 513 (D) 29 × 515 (E) 213 × 521
I 42020 Australian Mathematics Competition — Intermediate

18. Two sides of a regular hexagon are extended to


create a small triangle.
Inside this triangle, a smaller regular hexagon is
drawn, as shown.
In area, how many times bigger is the larger
hexagon than the smaller hexagon?
(A) 4 (B) 6 (C) 8
(D) 9 (E) 12

1 × 2 × 3 × 4 × 5 × 6 × 7 × 8 × 9 × 10
19. The number is a perfect square.
n
What is the smallest possible value of n?
(A) 7 (B) 14 (C) 21 (D) 35 (E) 70

20. In the triangle ABC shown, D is the A


midpoint of AC, E is the midpoint of
BD and F is the midpoint of AE.
If the area of triangle BEF is 5, what F D
is the area of triangle ABC?
(A) 30 (B) 35 (C) 40
E C
(D) 45 (E) 50

Questions 21 to 25, 5 marks each

21. A scientist measured the amount of bacteria in a Petri dish over several weeks and
also recorded the temperature and humidity for the same time period. The results
are summarised in the following graphs.

A B C D E
Bacteria

Humidity
Temperature

Week Humidity Temperature

During which week was the bacteria population highest?


(A) week A (B) week B (C) week C (D) week D (E) week E
2020 Australian Mathematics Competition — IntermediateI 5

22. Five friends read a total of 40 books between them over the holidays. Everyone read
at least one book but no-one read the same book as anyone else.
Asilata read twice as many books as Eammon. Dane read twice as many as Bettina.
Collette read as many as Dane and Eammon put together.
Who read exactly eight books?
(A) Asilata (B) Bettina (C) Colette (D) Dane (E) Eammon

23. There are 5 sticks of length 2 cm, 3 cm, 4 cm, 5 cm and 8 cm. Three sticks are chosen
randomly. What is the probability that a triangle can be formed with the chosen
sticks?
(A) 0.25 (B) 0.3 (C) 0.4 (D) 0.5 (E) 0.6

24. Five squares of unit area are circumscribed by a circle


as shown.
What is the radius of the circle?
√ √
3 2 5 10
(A) (B) (C)
2 3 2
√ √
13 185
(D) (E)
2 8

25. Alex writes down the value of the following sum, where the final term is the number
consisting of 2020 consecutive nines:

9 + 99 + 999 + 9999 + · · · + 99 


. . . 9 + 99 
. . . 9
2019 nines 2020 nines

How many times does the digit 1 appear in the answer?


(A) 0 (B) 2016 (C) 2018 (D) 2020 (E) 2021

For questions 26 to 30, shade the answer as an integer from 0 to 999


in the space provided on the answer sheet.
Questions 26–30 are worth 6, 7, 8, 9 and 10 marks, respectively.

26. If n is a positive integer, n! is found by multiplying the integers from 1 to n. For


example, 4! = 4 × 3 × 2 × 1 = 24.
What are the three rightmost digits of the sum 1! + 2! + 3! + · · · + 2020! ?
I 62020 Australian Mathematics Competition — Intermediate

27. A square of side length 10 cm is sitting on a line. Point P is the corner of the square
which starts at the bottom left, as shown. Without slipping, the square is rolled
along the line in a clockwise direction until P returns to the line for the first time.
To the nearest square centimetre, what is the area under the curve traced by P ?

P

28. Eight identical right-angled triangles with


side lengths 30 cm, 40 cm and 50 cm are ar-
ranged as shown.
The inner four triangles are made to over-
lap each other, but the outer four triangles
do not overlap any of the others.
What is the area, in square centimetres, of
the unshaded centre square?

29. My grandson makes wall hangings by stitching


together 16 square patches of fabric into a 4 × 4
grid. I asked him to use patches of red, blue,
green and yellow, but to ensure that no patch G B R Y
touches another of the same colour, not even di-
agonally. R Y G B
The picture shows an attempt which fails only
G B Y R
because two yellow patches touch diagonally.
In how many different ways can my grandson Y R G B
choose to arrange the coloured patches correctly?

30. A clockmaker makes a 12-hour clock but with the hour and
minute hands identical. An ambiguous time on this clock is
one where you cannot tell what time it is, since the exact 11 12 1
10 2
position of the two hands occurs twice in a 12-hour cycle.
9 3
For instance, the clock shown can be seen at approximately
8 4
7.23 pm and 4.37 pm so both of these times are ambiguous.
7 6 5
However, 12.00 pm is not ambiguous, since both hands are
together.
How many ambiguous times happen in the 12 hours from
midday to midnight?
Intermediate 2020 Answers
Question Answer
1 E
2 B
3 D
4 C
5 D
6 E
7 E
8 D
9 A
10 B
11 C
12 A
13 C
14 C
15 B
16 C
17 C
18 D
19 A
20 C
21 E
22 D
23 C
24 E
25 B
26 313
27 414
28 484
29 168
30 132
Australian Mathematics Competition
sponsored by the Commonwealth Bank
an activity of the australian mathematics trust

A u s t r a l i a n M at h e mat i c s T r u s t

NAME

YEAR TEACHER

2021
INTERMEDIATE DIVISION
AUSTRALIAN SCHOOL YEARS 9 and 10
TIME ALLOWED: 75 MINUTES

INSTRUCTIONS AND INFORMATION


GENERAL
1. Do not open the booklet until told to do so by your teacher.
2. NO calculators, maths stencils, mobile phones or other calculating aids are permitted. Scribbling paper, graph
paper, ruler and compasses are permitted, but are not essential.
3. Diagrams are NOT drawn to scale. They are intended only as aids.
4. There are 25 multiple-choice questions, each with 5 possible answers given and 5 questions that require a
whole number answer between 0 and 999. The questions generally get harder as you work through the paper.
There is no penalty for an incorrect response.
5. This is a competition not a test; do not expect to answer all questions. You are only competing against your
own year in your own country/Australian state so different years doing the same paper are not compared.
6. Read the instructions on the answer sheet carefully. Ensure your name, school name and school year are
entered. It is your responsibility to correctly code your answer sheet.
7. When your teacher gives the signal, begin working on the problems.

THE ANSWER SHEET


1. Use only lead pencil.
2. Record your answers on the reverse of the answer sheet (not on the question paper) by FULLY colouring the
circle matching your answer.
3. Your answer sheet will be scanned. The optical scanner will attempt to read all markings even if they are in
the wrong places, so please be careful not to doodle or write anything extra on the answer sheet. If you want
to change an answer or remove any marks, use a plastic eraser and be sure to remove all marks and smudges.

INTEGRITY OF THE COMPETITION


The AMT reserves the right to re-examine students before deciding whether to grant official status to their score.

©AMT Publishing 2015 amtt limited acn 083 950 341


2021 AMC
22 2021 AMC Intermediate Questions
Intermediate Questions

Intermediate
Questions Questions Division
– Intermediate
1. Each edge of this star is 2 cm long.
What is its perimeter?
(A) 5 cm (B) 10 cm (C) 15 cm
(D) 20 cm (E) 25 cm

2. The value of 2000 − 200 + 20 − 2 is


(A) 1778 (B) 1782 (C) 1818 (D) 1822 (E) 1888

3. What is the value of a in the diagram?


145◦
(A) 35 (B) 45 (C) 55
(D) 65 (E) 75 a◦

1
4. What is 50% more than ?
2
1 5 3 3
(A) (B) (C) (D) (E) 50.5
4 8 2 4

5.
1+3+5+7+9
=
2 + 4 + 6 + 8 + 10

1 5 11 9 63
(A) (B) (C) (D) (E)
2 6 12 10 256

6. Square ABCD has centre O. A B


The shaded area is 16 square units.
What is the length of the side of the square? O
(A) 4 (B) 8 (C) 16
(D) 32 (E) 64
D C

7. On the number line, which number is halfway between 102 and 104 ?
(A) 500 (B) 550 (C) 1010 (D) 2021 (E) 5050

www.amt.edu.au 22
2021 AMC
2021 AMC Intermediate Questions23
Intermediate Questions

8. To feed a horse, Kim mixes three bags of oats with one bag containing 20% lucerne and
80% oats. If all the bags have the same volume, what percentage of the combined feed
mixture is lucerne?
(A) 3 (B) 5 (C) 6 (D) 20 (E) 60

9. I have a solid block of wood in the shape of a cylinder. The top


and bottom faces meet the curved side at right angles. Suppose
that I slice the cylinder along a plane to create two smaller blocks
of wood.
Which of the following could not be the shape of the resulting
faces created by the slice?

(A) (B) (C) (D) (E)

10. Diya timed herself cycling laps around her suburb. After five laps, her stopwatch indicated
a time of 18 minutes and 15 seconds.
What was Diya’s average time per lap?
(A) 3 minutes and 3 seconds (B) 3 minutes and 15 seconds
(C) 3 minutes and 27 seconds (D) 3 minutes and 39 seconds
(E) 3 minutes and 51 seconds

11. I have four consecutive odd numbers. The largest is one less than twice the smallest.
Which of the following is the largest of the four numbers?
(A) 9 (B) 11 (C) 13 (D) 15 (E) 21

12. On a compact disc, uncompressed music data is stored as 44 100 samples for each second
of music, where each sample requires 4 bytes of data. Which of the following is closest to
the number of bytes required to store 5 minutes of music on the disc?
(A) 1 million (B) 5 million (C) 10 million (D) 50 million (E) 100 million

23
www.amt.edu.au
2021 AMC
24 2021 AMC Intermediate Questions
Intermediate Questions

13. In the figure, the value of x is x◦


3x◦
(A) 30 (B) 40 (C) 50
(D) 60 (E) 70 (x + 30)◦

14. What is the equation of the line passing through (0, 0)


that bisects the square in the diagram? 1
x x x
(A) y = (B) y = (C) y =
3 2 4
(D) y = 2x (E) y = 3x 1 2

15. A standard dice numbered 1 to 6 with opposite sides


adding to 7 is placed on a 2 by 2 square as shown.
The dice is rolled over one edge onto each of the four base
squares in turn and then back on to the original square,
as indicated by the arrows.
Which side of the dice is now facing upwards?

(A) (B) (C) (D) (E)

16. The two spinners shown are spun and the num-
bers that the arrows point to when they stop are
recorded. 6
1
3

For example, the numbers here are 3 and 6. 2 5


What is the probability that the sum of the two
numbers is even?
1 3 3 2 5
(A) (B) (C) (D) (E)
2 8 4 3 12

17. The area of the shaded region is given by c a



(A) ab + ac (B) a b2 + c2 a
b
(C) bc + a2 − ab − ac (D) ab + ac − bc
(E) ab + ac − a2 b
a
a c

www.amt.edu.au 24
2021 AMC
2021 AMC Intermediate Questions25
Intermediate Questions


18. If k and n are positive integers, and 10n + k = k, then the smallest possible value for k
is
(A) 3 (B) 4 (C) 5 (D) 6 (E) 10

19. Two squares are drawn as shown.


1
The smaller square covers of the larger square and
8
2
the larger square covers of the smaller square.
9
What is the ratio of the side length of the larger square
to the side length of the smaller square?
(A) 3:2 (B) 7:3 (C) 7:4
(D) 5:3 (E) 4:3

20. Six identical darts fit inside a regular hexagon as shown. Each
dart has three interior angles of 30◦ , and one of 270◦ .
What fraction of the large hexagon is shaded?
1 1 2 4 3
(A) (B) (C) (D) (E)
2 3 5 9 8

21. We want to place numbers into each of the blank squares in this
1 2
diagram so that each of the numbers we place is the average of
those in the squares directly connected to it.
What number should we put in the middle square of the top
row?
5 3 10 11 11 4 3
(A) (B) (C) (D) (E)
3 2 9 9 6

22. To set the timer on his microwave oven, Rick enters the digits of the hours, minutes and
seconds in order from left to right. For example, entering ‘12345’ sets the timer to 1 hour
23 minutes 45 seconds, while entering ‘408’ sets it to 4 minutes 8 seconds.
One day, Rick accidentally missed the last digit and the timer finished 4 minutes and 42
seconds earlier than he was expecting. What was the missing digit?
(A) 3 (B) 4 (C) 5 (D) 6 (E) 7

25
www.amt.edu.au
2021 AMC
26 2021 AMC Intermediate Questions
Intermediate Questions

23. I build a large cube from unit cubes. Then I completely paint a number of faces of the
large cube. When I dismantle the large cube, I find that I have 288 unit cubes without
any paint on them. How many faces of the large cube were painted?
(A) 1 (B) 2 (C) 3 (D) 4 (E) 5

     
1 1 1 1
24. The product 1− 2 1− 2 1 − 2 ··· 1 − 2 is equal to
2 3 4 15
7 8 9 10 13
(A) (B) (C) (D) (E)
13 15 16 21 24

25. Three artificial islands Razz, Sazz and Tazz were constructed in a shallow sea, each with
a coastline of 12 km.

3 4
Razz Tazz
3 Sazz 3 4 4
12
π 3

Around each island is a fishing zone, consisting of all points in the sea within 1 km of the
island. Which islands have a fishing zone of the largest area?
(A) Razz only (B) Sazz only (C) Tazz only
(D) Sazz and Tazz (E) All three have the same area

26. In Australian Rules football, a team scores six points for a ‘goal’ and one point for a
‘behind’. During a game, Vladislav likes to record his team’s score with a sequence of sixes
and ones. There are exactly three distinct sequences which give a final score of 7 points,
namely 6,1 and 1,6 and 1,1,1,1,1,1,1.
How many different sequences give a final score of 20 points?

www.amt.edu.au 26
2021 AMC
2021 AMC Intermediate Questions27
Intermediate Questions

27. What is the smallest natural number n such that the number

N = 100000 × 100002 × 100006 × 100008 + n


is a perfect square?

28. I have a large supply of matchsticks in four colours: red, yellow, blue and green. I use
them to make squares where each side is one matchstick long.
I count two squares as the same if one can be rotated and/or reflected to match the shape
and colour of the other.
How many different squares can be created?

29. Bluey divides the number 499 by each of the numbers 1, 2, 3, . . . , 499 and records the
remainders in order. So her sequence begins:

0, 1, 1, 3, 4, 1, . . .

Let M be the sum of these 499 remainders.


Jean-Luc divides the number 500 by each of the numbers 1, 2, 3, . . . , 500 and records the
remainders in order. So his sequence begins:

0, 0, 2, 0, 0, 2, . . .

Let N be the sum of these 500 remainders.


What is the difference between the numbers M and N ?

30. Tyler has a large number of square tiles, all the same size. He
has four times as many blue tiles as red tiles. He builds a large
rectangle using all the tiles, with the red tiles forming a boundary
1 tile wide around the blue tiles.
He then breaks up this rectangle and uses the tiles to make two
smaller rectangles. Like the large rectangle, each of the smaller
rectangles has four times as many blue tiles as red tiles, and the
red tiles form a boundary 1 tile wide around the blue tiles.
How many blue tiles does Tyler have?

27
www.amt.edu.au
2021 AMC
28 2021 AMC Senior Questions
Senior Questions

Senior Questions
Questions – Senior Division
1. Each small triangle is the same size.
What fraction of the largest triangle is shaded?
1 1 1
(A) (B) (C)
2 3 4
2 3
(D) (E)
5 8

2. When 2021 is divided by 7 the remainder is


(A) 2 (B) 3 (C) 4 (D) 5 (E) 6

3. What is 121 − 12−1 − 120 ?


11 1
(A) 0 (B) 1 (C) 10 (D) 11 (E) 11
12 12

4. In this diagram, what is the size of the angles marked θ


θ?
(A) 70◦ (B) 75◦ (C) 80◦
θ θ
(D) 85◦ (E) 90◦
150◦

(20 × 21) + 21
5. The value of is
21
(A) 20 (B) 21 (C) 22 (D) 41 (E) 42

6. Henry’s electric scooter took him 1.5 km in 3 minutes and 45 seconds.


What was the average speed of Henry’s trip in kilometres per hour?
(A) 20 (B) 21 (C) 24 (D) 25 (E) 30

83 × 36
7. =
65
(A) 6 (B) 48 (C) 72 (D) 128 (E) 256

www.amt.edu.au 28
Intermediate 2021 Answers
Question Answer
1 D
2 C
3 A
4 D
5 B
6 B
7 E
8 B
9 B
10 D
11 C
12 D
13 C
14 A
15 C
16 E
17 A
18 C
19 E
20 E
21 E
22 C
23 D
24 B
25 E
26 71
27 36
28 55
29 93
30 792

You might also like